You are on page 1of 69

1

Ch.1 Judicial Review


Judicial Review Judicial Review is the power of the federal courts to review acts of legislation. State Gov., executives, and other courts to determine whether it is consistent with the Constitution. Article III of the Constitution extended the judicial power to all cases arising under the Constitution. The constitution does not expressly authorize judicial review. Any state laws, state/federal statutes that are repugnant to the Constitution are unconstitutional. The constitution was adopted to limit the government. Supreme Court Original JD Article III of U.S. Const. granted the Supreme Court original jurisdiction in only 2 types of cases: 1. Those affecting ambassadors, other public ministers and counsuls 2. Those in which a suit between 2 or more states (exclusive) The SC JD is self-executing, congress has no power to neither add nor subtract from that JD. Supreme Court Appellate JD In other cases the SC only had appellate JD. 1. Appeals as of rightlosing party files an appeal, the court must hear and decide your case. 2. Certioraridiscretionary appeals, the court may decide to hear the case. The party losing in the highest state court may petition the SC to hear the case. Court decides the case only if 4 justices agree that the case is important. Brief are submitted, oral arguments are heard, court decides, and issues an opinion. (highest state court, all federal courts) Levels of Law Divine Law/Natural Lawdeemed the highest law Due Process of Lawidea that laws are concerned with liberty, substantive, and procedural due process. (gov. of law, not of man, gov. must obey and abide by the laws Fundamental lawslaws that are written down, shows importance

Marbury v. Madison Facts: Marbury sued to compel delivery of his commission as a Justice of the Peace after President Jefferson and Secretary of State Madison failed to deliver it to him after President Adams had appointed him. Chief Justice Marshall held that Section 13 of the Judiciary Act of 1789 was unconstitutional because it sought to confer on the Supreme Court original jurisdiction over a type of dispute over which the Constitution gave it only appellate jurisdiction. Where a statute violated the

Constitution it was the duty of the courts to apply the Constitution as paramount law which superceded inconsistent statutes. - Judiciary Act of 1789 Sec. 13The SC shall have appellate JD form the circuit courts and state courts and the power to issue writs of prohibitions and writs of mandamus to any courts appointed, persons holding office under the authority of the US. (Unconstitutional, cant expand SC original JD). Significance of Malbury v. Madison can be read as giving the judiciary power of judicial review or as giving the judiciary the ultimate power to interpret the Constitution

Review of State Acton The SC is authorized to review state decisions adverse to federal law, decisions that arise under the constitution. It is allowed because it creates uniformity between the states. (original/appellate) - Judiciary Act Sec. 25the final judgment or decree in any suit, In the highest court of law and equity of a state in which a court decision could be had, where it is drawn into question the validity of a treaty of statute, or an authority exercised under the US can be examined under the authority of the SC. Martin v. Hunters Lessee It was the first case to assert ultimate Supreme Court authority over state courts in matters of federal law. Section 25 of the Judiciary Act empowered the SC the power to review the decision of state courts that invalided a statute or treaty of the U.S. (federal law); Cooper v. Aaron Was a landmark decision of the Supreme Court of the United States, which held that the states were bound by the Court's decisions and had to enforce them even if the states disagreed with them. The State of Arkansas refused to be legally bound by the courts decision to desegregate school. Judiciary Act Sec. 25 upheld. Michigan v. Long The Supreme Court of the US is without JD to decide a case whose decision rests on state grounds. The SC requires a plain statement that a decision rest on adequate and independent state grounds, if so, they will deny certiorari. The SC wants to stay away from rendering advisory opinions.

Statutory Restrictions
Under the Exceptions Clause (Art. III, sec.2 cl. 2) Congress can make exceptions to and regulate the appellate jurisdiction of the Supreme Court. Congress is given the expressed power to make

exceptions to the SCs appellate JD. If a statute is enacted that gives the SC JD and is later repealed, the SC no longer has JD and must dismiss the case for lack of JD. No judgment can be rendered. (Ex Parte McCardle: held that Congress had power to remove the Courts appellate jurisdiction regarding habeas appeals conferred by an 1867 Act. Still, appellate jurisdiction remained as conferred by the Judiciary Act of 1789 so all routes to the Court were not eliminated. Article III gives federal courts the power to rule and decide cases. When a new law is clear that it is retroactive, the appellate court must apply that law in reviewing judgments still on appeal that were rendering before the law was enacted, and must alter the outcome accordingly. Judgments that are complete, courts cannot reopen final judgments. (Plaut v. Spendthrift Farm, Inc.)

Doctrine of Judiciability
Federal courts must confine their power of review to cases warranting judicial function. The Constitution delegated to federal courts the judicial power under Article III to only Cases and Controversies. - No advisory opinions - Must be ripemust have been harmed or immediate threat of harm - Mootness-must be a real, live controversyexcpetion, when issue of capable of repetition because of the short duration of the action, the controversy would not be moot. (pregnancy, elections) Advisory Opinions The Supreme Court can only decide cases where this is a case or controversy. An advisory opinion is an opinion in which there is no controversy but the parties request a legal opinion. The SC has no JD to hear those type of cases. It would be unconstitutional because it must be an issue that fits within the constitutional or some federal statute. (also applies to federal courts/judges). It interfere with the separation of powers. - Some state courts may give advisory opinions. Its usually based on that states constitution Muskrat v. US-illustrated the refusal to determine a constitutional issue absent an actual case. Congress authorized several Native American to sue the US to test the validity of a statute regarding the transfer of certain Indian Land. The party bringing a lawsuit must have Standing, i.e. it must show that it has suffered an injury in fact which was caused by the defendants conduct and that a favorable judicial ruling would redress that harm.

Political Questions
Political Questions put certain issues outside the bounds of judicial decision based on the subject matter involved. It does not withdraw all matters of political overtones from judicial review. Political questions relate to separation of powers concerns. A case that involves a political issue is not necessarily a political question. If it is a political question, the court must dismiss the case because they have no judicial power under Article III Sec. 2 because its not a case or controversy. - If the text of the Constitution explicitly defers a power/issue to another branch of Gov., then the court cannot decide the case. - Those inherently incapable of resolution and enforcement by the judicial process - Guarantee Clause Art. 4 Sec. 4US Constitution guaranty a republican form of government; a government that is decided by the people. Political Questions: 1. Questions regarding conduct of foreign relations; 2. Questions relating to which group of delegates should be seated 3. Procedures used by the senate to try impeachments Non-political Questions 1. Legislative apportionment 2. Presidential papers and communicationsare generally considered to be privileged and protected from disclosure in exercise of the executive power, when it an nonpolitical question when documents are necessary to continue criminal proceedings. The question of production is judiciable. Baker v. Carr--was a landmark United States Supreme Court case that retreated from the Court's political question doctrine, deciding that redistricting (attempts to change the way voting districts are delineated) issues present justiciable questions, thus enabling federal courts to intervene in and to decide reapportionment cases. Brennan reformulated the political question doctrine, identifying six factors to help in determining which questions were "political" in nature. Cases that are political in nature are marked by: 1. "Textually demonstrable constitutional commitment of the issue to a coordinate political department;" as an example of this, Brennan cited issues of foreign affairs and executive war powers, arguing that cases involving such matters would be "political questions" 2. "A lack of judicially discoverable and manageable standards for resolving it;" 3. "The impossibility of deciding without an initial policy determination of a kind clearly for nonjudicial discretion;" 4. "The impossibility of a court's undertaking independent resolution without expressing lack of the respect due coordinate branches of government;"

5. "An unusual need for unquestioning adherence to a political decision already made;" 6. "The potentiality of embarrassment from multifarious pronouncements by various departments on one question." Nixon v. U.S. 1993: POLITICAL QUESTION. Court held that the issue of whether the practice of a Senate Committee hearing evidence against an individual who has been impeached is constitutional under the Trial Clause (Art I, 3, cl 6) was non-justiciable issue b/c the power to try all impeachments is textually committed to Senate. Powell v. McCormack: NO POLITICAL QUESTION - court held that the constitution does not commit to Congress a blanket authority to determine whether to seat a member-elect, the courts are not barred from deciding the issue. Art I, 5 commits to Congress only the authority to judge the qualifications enumerated in the constitution. Under Art I 2 the House can only exclude elected representatives if they fail to meet the standing requirements of age, citizenship and residence so the House of Reps exclusion of Powell was unconstitutional. Gilligan v. Morgan: student at Kent University sued for injunctive relief against an Ohio national guard. The Plaintiff wanted a judicial evaluation of training weaponry and orders of the national guard. Because the plaintiff sought broad call on the judicial power to assume continuing regulatory JD over the activities of the national guard rather than a claim for past injuries or some other against a specific imminent threat, the case was nonjudiciable. The court had no JD, the constitution gave congress responsibility to prescribe discipline to the militia, and congress authorized the president to issue appropriate regulations. Political Gerrymandering: the courts have JD to decide apportionment cases. It is judiciable under the EP clause. Unconstitutional discrimination occurs only when the electoral system is arranged in an manner that will consistently degrade a voters or group of voters influence of the political process. It is the deliberate and arbitrary distortion of district boundaries and population partisan

Ch. 3 Commerce Clause/State Powers


10th Amendment Whereas the federal gov. only has those powers granted to it by the constitution. The state government of unlimited powers, having all powers not prohibited to them by the constitution. - Are not created by the constitution but have theirs source in sovereign state police power to regulate the health, safety, morals, and general welfare of the people. - The constitution limits the states power that interfere with individual liberty or powers delegated to the federal gov. - A valid act of congress or federal regulation supersedes any state or local action

Commerce Clause (Dormant CC) State Power 3 possible interpretations of the commerce clause 1. The power is exclusively federal and prohibit any and all atate regulations affecting interstate commerce 2. The power is concurrent in the state and federal government without any implied prohibitions on state power to regulate interstate commerce and is valid unless congress exercise its power to preempt state law 3. In the absence of federal regulation, the commerce clause may lie dormant, and invalidate some state laws affect commerce.

Dormant Commerce Clause Even when Congress has not acted, the dormant commerce clause subjects state legislation to review. Does a state law on a particcular subject deal with local issues or national issues. If local issue, the state has police power as long as it does not violate other constitutional clauses. If it is a national issue, then congress has power if it interferes with commerce between states. Federal Law is dormant if congress does not regulate a particular subject in which they have the power to regulate. QuestionAre states free to regulate in areas that may impact interstate commerce if congress has been silent on a specific issue? Dorman commerce clause scrutiny only arise when the state statute affects commerce and the federal government has not acted. - The textual analysis of the constitution is silent in this area - Courts have recognized that absent conflicting federal law that states retain some power to make laws governing matters of local concern, yet the states power is subject to constitutional limits imposed by the commerce clause even its in dormant state. - Judicial Actionwhen a court finds that a federal statute violates the commerce clause , it is rules unconstitutional based on violation of due process, congress cannot enact law to override it. When states offend the dormant commerce clause, the state strikes down the offending state measure. Congress can act under the commerce clause to endorse or override the courts decision under its dormant version. When can the state regulate? - If the state regulation does not intrude on the space the commerce clause covers, the Dormant Commerce clause does not restrict the state and it may act freely as long as 1. The regulation does not discriminate against out of state competition to benefit its local interest and 2. It is not unduely burdensome, the burden on IC does not outweigh the legitimate state interest.

It the state regulation falls within the federal commerce clause power, the question then arise is whether the state measure conflicts with any statute congress has enacted, if so the states action falls under the supremacy clause.

Early Cases Gibbons v. OgdenFulton invented the steamboat. NY granted Fulton the exclusive right to navigate on the Hudson River. Ogden was granted a license form Fulton to navigate boat on river btw NY and NJ. Gibbon began to run his boat btw the 2 states in defiance of NY monopoly. Ogden secured an injunction in NY restraining Gibbon from operating within NY waters. SC granted cert. - The court held that the NY law granting the monopoly was unconstitutional because it is repugnant to the US constitution, commerce clause and thus void. - Commerce extended beyond navigation to include commercial intercourse. - Regulate involved the power to prescribe the rule by which commerce could be governed. - Among the states did not include that commerce, which is completely internal, which is carried on between man and man in a State, or between different parts of the same State, and which does not extend to or effect other States. Implicitly, it did include commerce which affected another state even though it did not involve crossing a state line. National v. Local Matters Cooley v. Board of Wardens, 53 U.S. 299 (1852),[1] was a case in which the United States Supreme Court held that a Pennsylvania law requiring all ships entering or leaving Philadelphia to hire a local pilot did not violate the Commerce Clause of the Constitution. Those who did not comply with the law had been required to pay a fee. - "It is the opinion of a majority of the court that the mere grant to Congress of the power to regulate commerce, did not deprive the States of power to regulate pilots even though congress was silent on the subject. - The decisive criteria was the nature of the subject being regulated, the states freedom to act turned on whether the matter being regulated needed to be governed by national uniform measures - Here, national uniformity was not necessary, thus no violation. - SignificanceWhen Congress did not act the Commerce clause limited, but did not preclude the states ability to impact commerce. (national/local dichotomy). Test to determine whether law affecting commerce is unconstitutional. The Dormant Commerce Clause seeks to prohibit states from regulating in a way that discriminates against out-of-state competition. State laws that seek to accomplish economic protectionism are per se invalid.

1. If the law discriminates on its face, discriminates between in-state and out-of-state commerce, it will be struck down as economic protectionism, thus unconstitutional. (Strict scrutiny). 2. If the laws treat in-state and out of state equally, then the law must balance the state interest against the burdens on interstate commerce. 3. Least restrictive alternative approach --A state law that seemed discriminatory in its purpose or means can only survive if the state can show it has a legitimate purpose and not alternative means that was less burdensome on interstate commerce to achieve the purpose. Regulations Protecting Local Businesses Laws designed to protect local business interest against interstate competition is invalid. o Baldwin v. G. A. F. Seelig, Inc., 294 U.S. 511 (1935), is a United States Supreme Court case which held that a state may not regulate intrastate prices by prohibiting the importation of less expensive goods in interstate commerce. It established the principle that one state, in its dealings with another, cannot place itself in economic isolation. Regulations Requiring Local Operations If a state law requires a business to perform specific business operations in the state to engage in other business activity within the state, the law will be invalid as an attempt to discriminate against other states where the business operation can be performed more efficiently. o H.P. Hood & Sons v. Du Montthe state sought to protect its home industry against out of state competition when refused to license out of state milk distributer because NY feared loss of needed supply to the local market. Prohibition violated commerce clause. o Deans milk co. v. Madisonan ordinance made it unlawful to sell milk unless pasteurized and bottled within five miles of the town center. The court held that the ordinance discriminated against interstate commerce. The defended by citing safty reasons but the court found that it failed the least restrictive alternative approach because the city could have inspected the Ps plaint by local official or relied upon the finding of the local health official findings. Regulations Limiting Access to In-State Products A state law makes it difficult or impossible for out of state purchasers to have access to in state products is likely invalid. o Hughes v. Oklahomathe court stuck down a statute that forbade transporting minnows out of the state. Since the law discriminated on its face, Oklahoma has to justify it based on the legitimate purpose/least restrictive alternative test. The

state purpose was to conserve minnows was legitimate but its means discriminated against out-of-staters rather than adopted the less restrictive alternative such as placing a limit on capture. o Can prohibit in state owners from selling product out of state o A state cannot require an in state company to sell products at a lower price to instate residents than to out-of-state residents. Regulations prohibiting out-of-state waste o A state may not prohibit private landfill waste disposal facilities from accepting out of state garbage or waste or surcharge such waste. Exceptions 1. Necessary to an important state interest. A discriminatory state or local law may be valid if it furthers an important, non-economical state interest (health/safety) and there is no reasonable alternatives. o Maine v. Taylorstate can prohibit the importation of live baitfish in a state because the state could demonstrate that it had no way of effectively avoiding that the baitfish brings certain parasites into the state or had a detrimental affect on the states wild fish population. o However, state could not prohibit the export of baitfish to out of state purchasers because the sale of fish to the out-of-state customers would not impair any interst of the state, except interest of protecting local purchasers of baitfish from out of state competition. 2. Market Participant Exceptionthe CC does not prevent a state from preferring its own citizens when the state is acting as a market participant (buying, selling products, hiring labor, giving subsidies) o If the state acts as a market participant and not a regulator the dormant commerce clause does not apply and the states may favor its citizens. o The market participant exception applies when the state operates a business or purchase or sells goods or services (act as a business). o The language of the commerce clause imposes restraints on state regulation not participation in it. o Reeves v. StateSD owned and operated a cement plant. The state decided to supply SD customers before furnishing cement to others during the shortage. The discrimination was upheld because the Commerce clause only affected the states as regulators. o South-Central Timber v. Wunnickethe market participant exception does not insulate a state who seeks to regulate and participate in the market. The court struck down a state statute that required those who bought Alaskas timber to process it in-state. Can favor its residents but cant seek to regulate the later conduct of its customers.

10

3. The court is more lenient when the law favors government action involving the performance of a traditional government function. o United Hauler v. Oneidaholding that a New York state ordinance forcing private waste management companies to deliver waste to a public facility did not discriminate against interstate commerce. The Court held that because the facilities were owned and operated by a state the restriction was permissible because they treated every private business, whether in-state or out-of-state, exactly the samedo not discriminate against interstate commerce for purposes of the Commerce Clause." Nondiscriminatory Laws Balancing Test If the law does not discriminate on its face or in its purpose, the court weighs the importance of the states interest against the burdensome effects on interstate commerce. The least restrictive alternative must be used. 1. Less Restrictive Alternativecourts will consider whether the least restrictive alternative was used 2. Conflicts with other statesstate and local laws regulating commerce are more likely to be upheld when there is little chance that the regulation would have conflicting regulations of the same subject matter o South Carolina Highway Dept. v. Barnwell Brosthe court upheld a state law prohibiting the use of trucks wider than 90 inches and heavier than 20,000 lbs. The law was aimed at both safety and highway conservation. A state may impose non-discriminatory restriction with respect to the character of motor vehicles moving in interstate commerce as long as it aimed at safety and economical use of highway. o Southern Pacific Co. v. Arizonademonstrates that state laws might violate the Commerce Clause even when in-state and out-of-state commerce are treated equally. Arizona prohibited operation of a train longer than 70 freight cars. It was defended on grounds of a safety measure. The court held that the statue placed too heavy of a burden on interstate train operation and that the safety advantages did not outweigh the cost to interstate commerce.

Privilege & Immunities Clause


The privilege and immunities clause states that the citizens of each state shall be entitled to all privileges and immunities of citizens in the several states. Whatever rights, privileges granted to own citizens shall be imposed on citizens of other states within the states JD. Privilege and Immunities of State Citizenship (Article IV) 1. Unlike dormant clause, it extends only to individuals, not corp.

11

2. Only fundamental rights are protectedinterstate P&I clause does not prohibit all discrimination by a state in favor of its own citizens, but only when it denies a fundamental right (social and civil liberties or means to the non-residents livelihood). The threshold issue is whether an activity is a Privilege and Immunity. These relate to activities which are sufficiently basic to the livelihood of the Nation. These would include the right to police and fire protection when out of state, the right to medical care, the right to pursue a trade and the right to engage in political speech and religious worship. (Baldwin v. Fish & Gamerecreational activity does not qualify as a privilege or immunity like elf hunting as a recreational sport, not commercial. Unlike shrimping which is commercial, not done for recreation.) o Hicklin v. Orbeckthe legislature passed an act which required all oil and gas to which the state is a party to include provisions in preferential treatment in hiring Alaskian residents over nonresident. The act violated the privilege and immunities act Art. 4 Sec. 2 and the equal protection of the 14th amendment. Discrimination against nonresidents of a state is only allowed when the nonresident constitute a peculiar source of evil at which the statute is aimed to protect. o United Building and Const. v. City of Camden-- A municipal ordinance of the city of Camden, New Jersey, required at least 40% of the employees of contractors and subcontractors working on city construction projects to be Camden residents. Refer to Hicklin 3. Exceptiona state law discriminating against nonresidents may be valid if the state has a substantial justification for the different treatment. It must show that the nonresident is part of the problem or caused the problem, and there is no less restrictive means to solve it. And the challenged law must bear a substantial relationship to that interest. Then the least discriminatory measures that could have achieved the same end must be used. 4. Market participant doctrine does not apply to privilege and immunities clause, states cannot insulate its actions on grounds that it was acting as a market participant, not a regulator. The Privileges & Immunities Clause is an actual Constitutional text to protect peoples rights. Thus, since the clauses have two distinct purposes, the market participant exception did not apply to the Privileges and Immunities analysis.

14th amendment privileges and Immunities


Prohibits states from denying their citizens the P&I of national citizenship, such as right to vote for federal offices, right o travel, right to DP. 1. Corporations are not protected 2. Bill of rights not includedSlaughterhouse cases, held that fundamental rights protected against federal abuse (first 10 amendments) are not privilege and immunities of national citizenship within the meanings of the 14th amendment; nor is right to work, live, eat. Thus the guarantee of the bill of rights are protected from state action only by DP and EP of the 4th amendment

12

3. Right to travel--protected by the 14th amendment, included the rights of newly arrived citizens to enjoy the same privileges and immunities a are enjoyed by other citizens of the state. Saenz v. Roe

Ch. 4 Federal Powers under Com. Cl.


Congressional Powers
Powers Expressed Most powers of Congress are set forth in Article I section 8 (power to set taxes, regulate commerce, rules of naturalization). Implied Powers (Necessary and Proper Clause) Article I Section 8 cl. 18 states that if necessary, congress can make all laws which are necessary as a means to an end to affectuate one of its powers. ( the N&P clause is not itself a power, it gives congress the power to execute specifically granted powers 5. In McCulloch v. Maryland, (1819), Chief Justice Marshall held that the Constitution implicitly authorized Congress to take the means necessary to give effect to the powers granted. Congress concluded that a national bank was necessary and proper. 6. Necessary does not have to mean absolutely necessary as long as congress believes that it is necessary. Proper establishes that the end/goal needs to be legitimate, that is it within the scope of the constitution (a part of the expressed powers in section 8). 7. Let the end be legitimate, let it be within the scope of the constitution, and all means which are appropriate, which are plainly adapted to that end, which are not prohibited, but consist with the letter and spirit of the constitution are constitutional. Commerce Clause The commerce clause on its face, vest congress the power to regulate commerce. Article 1 Sec. 8 states congress shall have the power to regulate commerce with foreign nationals, and among several states, and with the Indian Tribes. 8. Creates a national market which the state cannot destroy by advancing their own economic interest. 9. It helps to foster national political cohesion by inhibiting states from imposing reciprocal barriers that would further divided the states

Three Broad Categories that Congress may regulate under the ICC 1. Congress may regulate the use of the channels of interstate commerce; 2. Congress is empowered to regulate and protect the instrumentalities of interstate commerce, or persons or things in Interstate Commerce, even though the threat may come only from intrastate activities;

13

o Things or buildings that operate in and promote the flow of people and goods through the channels of interstate commerce such as airplanes, cruise ships, gas stations, airports, seaports, restaurants, hotels. 3. Congress's commerce authority includes the power to regulate those activities having a substantial relation to interstate commerce (i.e., those activities that substantially affect interstate commerce). o the court would uphold the regulation if its of economical or commercial activity, and there is a rational basis in which congress can conclude that in the aggregate its substantially affects interstate commerce. If regulating intrastate activity is noneconomic or noncommercial it cannot be regulated under the commerce clause unless congress can show a substantial effect of interstate commerce o congress can regulate intrastate non-economical activity within the scope of its regulation of a larger economical activity. The court may consider the aggregate affect of the individual conduct that affects interstate commerce or regulation (drug trade)(rational basis standard) o Violent crimes that are tied to interstate commerce can be regulated (under Morrison, violent crimes standing alone are the type of noneconomic activity that congress cannot regulate. o In Lopez and Morrison-the court focused narrowly on the specific conduct set forth in the statute. Lopez-possession of a firearm, Morrison-gender motivate violence, neither case involved an economical exchange or transaction. Economical activity is production, distribution, or consumption of a good or service, employment o Congressional findings will not support regulation of an intrastate activity when the findings are based on an attenuated connection to interstate commerce. Pre-1937 Cases Productive Industries & Commerce For nearly a century thereafter Gibbons, the Court's Commerce Clause decisions dealt but rarely with the extent of Congress' power, and almost entirely with the Commerce Clause as a limit on state legislation that discriminated against interstate commerce.[9] Under this line of precedent, the Court held that certain categories of activity such as "production", "manufacturing", and "mining" were within the province of state governments, and thus were beyond the power of Congress under the Commerce Clause.Control over product was left to the states. Formalistic approach: (Production v. Commerce) (direct v. Indirect) (local v. National) 10. In The Daniel Ball (1871), Congress passed an Act in 1838 prohibiting the operation of steam ships on the internal navigable waters of the United States without a license. The Daniel Ball was fined $500 for operating without the license. The Supreme Court held that Congress has the power to regulate the intrastate transport of goods if those goods are bound for or originated from another state.

14

11. Perez v. United States(Loan Sharks) Petitioner was convicted of "loansharking" activities, i.e., unlawfully using extortionate means in collecting and attempting to collect an extension of credit, in violation of Title II of the Consumer Credit Protection Act, and his conviction was affirmed on appeal. He challenges the constitutionality of the statute on the ground that Congress has no power to control the local activity of loansharking. Title II of the Consumer Credit Protection Act is within Congress' power under the Commerce Clause to control activities affecting interstate commerce, and Congress' findings are adequate to support its conclusion that loansharks who use extortionate means to collect payments on loans are in a class largely controlled by organized crime with a substantially adverse effect on interstate commerce. Even though activity is purely intrastate, it may be in the judgment of congress that it affects interstate commerce. 12. Manufacturing 13. Manufacturing is not a part of commerce, it deemed beyond congressional power 14. Direct v. IndirectThe court does have the power to control manufacturing of a given thing if it affected commerce directly but not indirectly. a. U.S v. E. C Knight (1895)the defendant obtained a monopoly over the manufactured sugar. The Gov. alleged that the Defendant violated the Sherman Act by acquiring its principal competitors. The SC held that congress could not intervene and had no manufacturing power under the commerce clause. 15. Just because a commodity is manufactured or produced within a state and is intended for interstate commerce, does not mean that its production or manufacturing is subject to federal regulation under the commerce clause. 16. A commodity that is meant to be sold in interstate commerce is not considered to be part of interstate commerce before the commencement of its movement from the state. a. Carter v. Carter Co.-- Mining is not interstate commerce. It is a local business and is subject to local control and taxation. The Gov. could not regulated prices because labor provisions affect production not commerce. i. Delegation of Leg. Power-- invalidated regulations of the poultry industry according to the nondelegation doctrine and as an invalid use of Congress's power under the commerce clause. Section 3 of the National Industrial Recovery Act empowered the President to implement industrial codes to regulate weekly employment hours, wages, and minimum ages of employees. The codes had standing as penal statutes. The Court held that Section 3 was "without precedent" and violated the Constitution. The law did not establish rules or standards to evaluate industrial activity. In other words, it did not make codes, but simply empowered the President to do so. The Court held that the codes violated the constitutional separation of powers as an impermissible delegation of legislative power to the executive branch. The Court also held that the NIRA provisions were in

15

excess of congressional power under the Commerce Clause. The Court distinguished between direct effects on interstate commerce, which Congress could lawfully regulate, and indirect, which were purely matters of state law. Though the raising and sale of poultry was an interstate industry, the Court found that the "stream of interstate commerce" had stopped in this caseSchechter's slaughterhouses bought chickens almost exclusively from intrastate wholesalers and sold completely exclusively to intrastate buyers. Any interstate effect of Schechter was indirect, and therefore beyond federal reach. b. In Hammer v. Dagenhart (1918), Congress passed the Child Labor Act in an attempt to combat the use of child labor in factories. The Supreme Court held that Congress did not have the power under the Commerce Clause to regulate goods produced through child labor and transported in interstate commerce. The Court held that manufacture is not commerce and the exclusion of goods was permitted only when it involved the nature of the goods themselves, not the manner in which they were made. (overruled by Darby)

New Deal Commerce Clause Cases/Regulation of Local Activity The increase of Congress powers under the Commerce Clause. When examining whether some activity was considered "Commerce" under the Constitution, the Court would aggregate the total effect the activity would have on actual economic commerce. Intrastate activities could fall within the scope of the Commerce Clause, if those activities would have any rational effect on Interstate Commerce. Finally, in United States v. Darby Lumber Co., 312 U.S. 100 (1941), the Court said the 10th Amendment "is but a truism" and was not considered to be an independent limitation on Congressional power. Just because the state can pass the law under state police power, doesnt mean that congress cant pass the same law under the commerce clause or necessary and proper clause. 17. Although activities may be intrastate in character when separately considered, if it has a close and substantial relation to interstate commerce that their control is essential or appropriate to protect that commerce from burdens and obstructions, congress has the power to exercise control. 18. The federal government's power to regulate interstate commerce also allowed it to regulate purely intrastate commerce in cases where control of the former was not possible without control of the latter. (close and substantial relationship to interstate commerce) a. The Shreveport Caseupheld congressional authority to regulate intrastate rail rates that discriminated against interstate rail carriers. The court reasoned that congress can regulate intrastate rail activities that had a close and substantial relationship to interstate rail traffic.

16

19. Production/manufacturing could on occasion be subject to congressional regulation. No longer adhere to the direct/indirect affect on interstate commerce. (close and substantial relation to commerce) Although activities may be intrastate in character when considered separately, if it has a close and substantial relation to interstate commerce that their control is essential or appropriate to protect that commerce burdens and obstructions, congress has the power to regulate it. a. In NLRB v. Jones & Laughlin Steel Corp. (1937), Congress enacted the National Labor Relations Act of 1935 which prohibited unfair labor practices affecting interstate commerce. Jones & Laughlin Steel was charged under the Act for allegedly discriminating against union members. The Supreme Court held that Congress has the power to regulate manufacturing activities that have a significant effect on interstate commerce, including activities that burden interstate commerce or its free flow. 20. Congress could regulate productive activity (intrastate) that had a substantial effect on commerce. Regulated intrastate activity need not self impact commerce. Rather, congress can reach local activities with trivial effects on commerce if the cumulative effect of such activities when aggregated was weighty. a. In United States v. Darby Lumber Co. (1941), Congress passed the Fair Labor Standards Act of 1938 establishing a minimum wage and maximum hours for employees involved in producing goods for interstate commerce. The Supreme Court upheld the statute, holding that Congress can exclude from interstate commerce articles which deteriorate the health, welfare, and morals of the nation. o Wickard v. Filburn (1942 allowed Congress to regulate farm production intended solely for consumption on the farm. The cumulative effect of such production, when aggregated could have a substantial effect on commerce, thereby justifying federal regulation. The power to regulate commerce includes the power to regulate prices, which in turn includes the power to regulate practices that affect prices, which in turn includes the power to regulate production b. Court upheld a Civil Rights Act prohibited private racial discrimination in any place of public accommodation if its operation affects commerce, (Heart of Atlanta Motel). Statute also applied to family owned restaurants , not because the customers were interstate travelers but because substantial portion of it food traveled in interstate commerce before being purchased from local suppliers. (Katzenbach v. McClung). i. Regulatory legislation affecting ordinary commercial transaction shall not be pronounced unconstitutional unless the state law (1) burdens fundamental rights (2) unless it restricts political process (3) when the state law harms discrete and insular minorities. (gov. must come up with better justification for laws)

17

Limits on the Federal Gov. Powers under the CC the proper test requires an analysis of whether the regulated activity substantially affects interstate commerce. the substantial effect must be an economic effect, and that as far as Congress powers might extend under the Commerce Clause, the regulation must somehow involve commerce. o The SC invalidated the Gun Free School Zone Act which had made it a federal offense to knowingly possess a firearm in a school zone. The statute had nothing to do with commerce so congress had no police power. Congress could only regulate intrastate activity that substantially affected interstate commerce, but the underlying regulated activity must be commercial or economical. (U.S v Lopez) o The court invalidated a provision of the violence against women act that had created a federal civil remedy for victims of violent crimes that were motivated by gender bias. The court held that the gender-motivated crime is not economic or commercial activity, thus not commerce. (U.S. v. Morrison). o 10th amendment-- congress cannot compel a state to enact a federal regulation. (Printz v. United States, New York v. United States) The government cannot compel a state to enact a federal regulation that is noneconomic and not commerce, or does not affect commerce The Commerce Clause will support federal regulation of commercial or economic activity which has a substantial effect on interstate commerce or which in the aggregate has a substantial effect on interstate commerce, but the effects of noneconomic activity cannot be aggregated this way in order to fall under Commerce Clause power. Rational Basis Review - The evolving level of scrutiny applied by Federal courts to Commerce Clause cases should be considered in the context of rational basis review. The Supreme Court has exercised little independent judgment, choosing instead to defer to the expressed or implied findings of Congress to the effect that regulated activities have the requisite "economic effect". - Of course, the mere fact that Congress has said when particular activity shall be deemed to affect commerce does not preclude further examination by this Court. But where we find that the legislators, in light of the facts and testimony before them, have a rational basis for finding a chosen regulatory scheme necessary to the protection of commerce, our investigation is at an end.

Tenth Amendment (Limits on Federal Power)


Whereas the federal gov. only has those powers granted to it by the constitution. The state government of unlimited powers, having all powers not prohibited to them by the constitution.

18

Are not created by the constitution but have theirs source in sovereign state police power to regulate the health, safety, morals, and general welfare of the people. The constitution limits the states power that interfere with individual liberty or powers delegated to the federal gov. A valid act of congress or federal regulation supersedes any state or local action the Constitution grant to Congress authority to regulate Commerce among the several States and the Constitution grant to Congress of authority to make all Laws which shall be necessary and proper for carrying into Execution the foregoing powers

4 conditions must be satisfied before a state activity may be deemed immune from a particular regulation under the commerce clause. o The federal statute at issue must regulate the state as states o The statute must address matters that are indisputably attributed to states sovereignty o The states compliance with the federal regulation must directly impair the states ability to structure integral operations in areas of traditional state functions o The relation of state and federal interest must not be of such that the nature of federal interest justify state submission. Federal government can use the commerce clause to regulate traditional government function. There are 2 limits on the 10th amendment limits on federal powers. 1. Congress cannot commander state to make federal laws/regulation 2. Nor command executive official to enforce or implement federal laws. o CommanderingNew York v. United Statesheld that congress cannot coerce states into making law by forcing them to enact one of two laws that neither of which congress can direct to act alone. Ask to either enact federally proscribed regulation on radioactive waste or accept liaibly for waster, even privately owned within its borders. Violation of the 10th amendment because congress could not force the states to enact the regulation standing along so could not force to choose between the 2. o Commanding executive official to enforce lawsPrintz v. United States--the Court again ruled that the Brady Handgun Violence Prevention Act violated the Tenth Amendment. The act required state and local law enforcement officials to conduct background checks on persons attempting to purchase handguns. The law violated the Tenth Amendment since the act "forced participation of the State's executive to exercise their sovereign power to enforce the regulation. Difference between commanding and requiring a state to comply with general acceptable federal laws. Commandering occurs when the federal regulation directs the state leg, to enact laws/provisions or directs the state

19

officials to exercise sovereign power over private persons. General compliance with federal law does not require states to enact any federal laws or regulations. It only requires actions to be taken to ensure compliance. th The 10 amendment is not violated if the federal government gives the state the choice to enact law or they would withhold spending, can use spending power, however it cannot be coercive such as withholding the entire amount, that would be the equivalent of forcing to enact law. The 10th amendment is not violated if the federal government gives the choice to either enact the law or the federal government will preempt it by use of commerce power. Congress should have passed a federal regulation under the necessary and proper clause , commerce clause, or taxing and spending power. However, congress cannot compel a state to enact a federal regulation. Congress can regulate state employees under a general statute that covers both private and public employees. Congress has the power under the commerce clause to extend the Fair Labor Standard Act to employees of the state and local government (originally it only applied to private companies), which required employers to provide minimum wage and overtime pay. Garcia v. San Antonio (congress defines what a traditional government function is, not the court)

PreemptionCongressional Conflict The Congress may exclude state regulation in a particular area. 1. Express/conflict between federal and state lawa state statute is void if it conflicts with a valid federal law. (supremacy clause). 2. Express/Fieldstate regulation must fail if congress enacted with the intent to preempt an entire field of regulation. (congress have left no room for supplementary state regulation). In a field traditionally within the power of the states (health, safety, welfare) it is presumed that the state police power are not superseded unless there was a clear and manifest purpose of congress. 21. When the field of regulation is one the states traditionally occupy, the police power of the states should not be lightly superseded. (presumption). 22. Silkwood v. Kerr-McGee Corp.

Taxing and Spending Power


Article I, Section 8, Clause 1 of the United States Constitution, is known as the Taxing and Spending Clause. It is the clause that gives the federal government of the United States its power

20

of taxation. The powers to tax and spend are concurrent powers of the federal government and the individual states. Typically, the power is used to raise revenues for the support of government. But, Congress has employed the taxing power in uses other than solely for the raising of revenue, such as: (1) regulatory taxation taxing to regulate commerce (2) prohibitive taxation taxing to discourage, suppress, or even exterminate commerce. Also, congress not the court, determines if taxing is appropriate for the general welfare. Two different views on Taxing and Spending Power: - Madison interpreted the clause narrowly to mean that congress was limited to taxing and spending only in the exercise of one of its enumerated powers. (not the standard) - Hamiltonintrepreded the clause broadly as the power to tax and spend for any purpose congress believed served the general welfare.

Taxing Power (Federal Taxation on State or Local Government)


Modern Taxing Power - Federal taxes will be upheld under the Taxing and Spending Clause if it raises revenues or intended to raise revenue regardless of ulterior legislative motive of Congress to regulate. A tax is not invalid merely because it regulates, discourages, or deters the activity taxed, as long as it generates revenue. - Under the taxing power of Congress, it allows congress to reach all matters beyond its direct regulatory authority. (even intrastate production). Still it may not be exercised in violation of specific constitutional rights. - Chas C. Steward Machine v. Davisreasons why taxing is not unconstitutional (1) uniform (2) not a violation of 5th amendment due process (3) 10th amendment not violated because taxing and spending clauses is delegated to the federal government (4) does not call for surrender of the states power. Power of the state to tax interstate commerce Pursuant to the CC, congress has complete power to authorize or forbid state taxation affecting interstate commerce. If congress has not acted, look to see whether the tax discriminates against interstate commerce. If it does it not valid. If it does not, assess whether the tax burden on interstate commerce outweighs it benefit to the state 1. There must be a substantial nexus between the tax payer and the state 2. The tax must be fairly apportioned 3. There must be a fair relationship between the tax and the services or benefits provided to the state. - Discriminatory taxes not permitted, may make a few arguments o Tax discriminates on nonresidentsP&I o EPcongress can give state power to violate commerce clause, but cannot authorize state to violate EP

21

Taxing or regulation that only applies to the state and private entities The court will not strike downon 10th amendment groundsa tax or regulation that subject states or local governments to regulations or taxes that apply both to the public sector and private sector. (Garcia v. san Antonio Transit Authoritycongress required a local and state government to follow the provisions of the Federal Fair Labor and standard Act requiring minimum wages for all employees). Can congress under its tax power, regulate an activity beyond its power otherwise? - Congress may impose a true tax (to raise revenue) on activity that it cannot otherwise regulate, however cannot impose a penalty tax. o Bailey v. Drexel Furniture Co., 259 U.S. 20 (1922), Congress sought to use its taxing power to regulate what the court said congress could not do under the commerce clause. The court had previously invalidated as beyond the federal commerce power a federal law barring products made by child labor. Congress then attempted to tax profits of employers of children. The tax on child labor was unconstitutional because it was a penalty, not a tax, on employment of children. This is no longer good law. Congress can tax unfavored behavior. o United States v. Kahriger-- held certain provisions of the Revenue Act of 1951 were constitutional, in particular sections related to an occupational tax on persons involved in gambling. The Supreme Court ruled that the Congressional purpose of penalizing interstate gambling under the guise of imposing a tax did not violate the Constitution by infringing the police power reserved to the states. Also, it produces revenue. o United States v. Butlerthe Agriculture Adjustment Act sought to restrict farm production through incentives. This act was intended to regulate agricultural production, a matter which was deemed within the control of the state. Because congress could not directly regulate agriculture, it could not indirectly do so under its taxing power. No longer good law. Can tax.

Spending Clause
Congress shall have the power to pay the debts and provide for the common defense of the general welfare if the US. Congress has broad discretion to decide which expenditures will promote the general welfare. - Court adopted the Hamilton Approach - In addition to the power to use taxes to punish disfavored conduct, Congress can also use its power to spend to encourage favored conduct. - Congress can use spending power to bribe the states into doing what they could not regulate. In South Dakota v. Dole, the Court upheld a federal law which withheld highway funds to states that did not raise their legal drinking age to 21. The

22

Congressional condition of highway funds is merely a "pressure" on the State to comply, not a "compulsion" to do so, because the State's failure to meet the condition deprives it of only 5% of the highway funds it may obtain. Therefore, Congress has not run afoul of the Tenth. Even if congress lack the power to impose a national minimum drinking age directly, encouragement to state action found in the federal law is a valid use of the spending power. o Conditional Spending: Condition must be stated Condition must serve the general welfare Condition must be reasonably related to the purpose in which federal funds are allocated. Condition does not cause state to violate independent constitutional bars. If congress has an independent power to regulate, then it can spend under the necessary and proper clause whether they have actually regulated in that field or not. If there is no independent power to regulate, then the spending must be for the general welfare and not have a local purpose. Congress cannot coerce a state to act opposed to encouraging them to do so (withholding 100% of federal funds) Congress can use it spending power to indirectly enact laws (by encouraging states to do so) that they could not enact directly Congress can use enumerated powers to regulate traditional state functions by spending powers Helvering v. Davis--which held that Social Security was constitutionally permissible as an exercise of the federal power to spend for the general welfare, and did not contravene the 10th Amendment. The Court defended the constitutionality of the Social Security Act of 1935, requiring only that welfare spending be for the common benefit as distinguished from some mere local purpose.

Intergovernmental tax immunity


1. No direct tax on Federal Instrumentalitiesa state cannot tax property or operations of the federal government. No discriminatory tax on federal employees 2. Non-discriminatory, indirect tax on the federal government or its property is permissible, as long as its not unreasonably burdensome. (can tax federal employees, but cannot impose a higher tax on federal employees than on local and state employees, can tax all employees evenhandedly).. May not impose a higher tax on federal employees 3. The states have no power to regulate the activities of the federal government. The instrumentalities and agents of the federal government are immune from state regulation/taxation.

23

4. A state must apply the same tax treatment to federal employees as it does to state employees. (if exempt from state tax must be exempt from federal tax).

Congressional Powers
Citizenship The 14th amendment makes citizenship dependant solely upon place or birth or naturalization. Such persons are citizens of the United States and the states in which they live. Congress cannot alter the constitutional effect of birth within the US. Entitlements 1. Aliensonce in the US entitled to due process and equal protection 2. Personcan claim due process and equal protection 3. Citizendue process, equal protective, and all privileges and immunities Congress Power Over Citizenship Congress can establish a uniform rule of naturalization. Have plenary power over aliens Exclusions of aliens - Nonresident alienshave no right to enter the US, can be refused entry based on political beliefs - Resident aliensthey are entitled to notice and hearing before they can be deported Naturalization and denaturalization - Congress cannot take away citizenship of a citizen native born or naturalizedwithout consent - A citizen can relinquish citizenship by words of expression or conduct - A person born in another country to a UC citizen does not have a constitutional right to become a US citizen. Congress can condition citizenship of children born abroad on their return to live in the US within a specific period of time or specific number of years. Citizen/Naturalization/Termination of Citizenship Citizenperson born or naturalized in the US Naturalization-- Congress has power to fix the conditions in which naturalization is to be granted or withheld. It prescribes the rules in which aliens can become citizens. Congress may grant citizenship to named individuals or provide for collective naturalization. Expatriationabsent an involuntary renunciation, congress lacks the power to remove citizenship.

24

Afroyim v. RuskAmerican citizen was refused a passport on the grounds that he had lost his citizenship by voting in a foreign election. Court ruled that congress did not have that power because voting is not a voluntary renunciation. Once acquired, this 14th amendment citizenship was not to be shifted, cancelled, or diluted at the will of the federal government, the states, or any other government unit. Congress has broad power over the admission and deportation of aliens. The conditions of entry of every alien, the particlaur classes or aliens that shall be denied entry all together, the basis for determining such classifications, the right to terminate such hospitality to aliens, the grounds on which such termination shall be based, have been recognized solely for yhr responsibility of congress and is outside the power of the courts. o Galvin v. Pressdue process was not violated where congress ordered the deportation of an alien who has legally joined the communist party. o Fiallo v. BellCongress can discriminate against an alien in a way that they cannot discriminate against citizens. Alien fathers of U.S. citizens are granted special immigration preference. Unwed fathers of illegitimate offspring are denied the preference. The condition for entry of every alien, particular classes of aliens that should be denied entry all together, the basis for such determination of classification, the right to terminate hospitality to aliens, the grounds at which the termination should be based, have been recognized as matters solely for the responsibility of congress and wholly outside the power of the court. o United States v. Knauffalien wife was deported because the attorney general found that her admission would be prejudicial to the interest of the US. Whatever the rule may be concerning the deportation of a person having gained entry into the US, it is not within the province of the court, unless expressly authorized by law, to review the determination of the political branch to exclude a given alien..whatever the procedure authorized by congress is, it is due process as far as an alein denied entry is concerned. o Rogers v. BelleiCongress may provide that a person, who acquires citizenship by virtue of having been born abroad to parents one of whom is a US citizen, shall loose citizenship unless he resides in the US continuously fro 5 yrs between the ages of 14 and 28. Congress may constitutionally enact a statute imposing conditions for citizenship upon those whose citizenship is created outside the parameters of the Fourteenth Amendment. The Supreme Court will deem such a statute valid if it fits within the restrictions of constitutional provisions other than the Fourteenth Amendment, and if the statutes conditions are not unreasonable, arbitrary, or unlawful.

25

Delegation of Legislative Power Congress has power under Art. I sec. 8 to delegate its authority to another part of the government so long as congress have made it clear what they have delegated. They cannot delegate their law making powers. Cannot delegate legislative authority. Congress cannot choose what legislative prerogatives to delegate away. - Congress have broad power to delegate its legislative power to executive officers and administrative agencies. - Congress can even delegate rule making power o Mistretta v. United StatesCongress can delegate the power to establish sentencing guidelines for criminal cases to a sentencing commission located in the federal courts and must up in part of federal judges as long as the tasks delegated do not undermine the integrity of the judiciary or usurp the powers of other branches. - Limitation o The power cannot be uniquely confined to congress (power to declare war, impeach) o Clear standardincludes intelligible standard for the delegate to follow o Separation of powers limitationcongress cannot give itself power to remove an officer of the executive branch by means other than impeachment; it cannot delegate rule making power to an executive branch then reserve right to fire agency head; congress cannot give a government employee who is subject to removal by congress purely executive powers (no legislative veto)

Delegation Doctrine (Foreign Powers) - Delegation Doctrine applies to domestic policy but may delegate powers to the president when it comes to foreign affairs. - The parameters of the constitutionality of the Presidents legislative power differ when external, rather than internal, affairs are at stake. For international affairs, the President does not require a congressional act to establish legislative authority in foreign relations. United States v. Curtiss-Wright Export Corp. - The US Gov. has inherent powers when it comes to foreign/international issues that is not proscribed in the constitution. It has inherent powers because it is a state/gov according to international law

Legislative Veto
The legislative veto represents congress efforts to delegate certain powers to the executive while retaining some control over its decisions. Legislative vetos of executive actions are invlaid. It

26

usually arises when congress delegates discretionary power to the executive branch, then attempt to control the delegation by reserving for itself a veto provision. In order to valid, the legialtive action must be approved by both houses and be presented to the president to sign. - Presentment clauseevery bill that passes the house and the senate must be presented to the president for approval - Bicameralismno law can take effect without the prescribe majority of the house and the senate - Separation of powersthe one house veto overruled the duty that was delegated to the executive branch. That duty cannot be revoked until it is legislatively altered by enacting legislation that passes through bicameralism and presentment or revoking the statute that delegated the power. - When the framers intended to authorize Congress to exercise powers outside of bicameralism it explicity provided for it o HORimpeachment, Senateconduct impeachment powers, Senate approve/disapprove presidential appointments, Senateratify treaties.

Congressional Privileges/Speech or debate clause


Article I, 6, Cl.1 The clause states that members of both Houses of Congress...shall in all Cases, except Treason, Felony, and Breach of the Peace, be privileged from Arrest during their attendance at the Session of their Respective Houses, and in going to and from the same; and for any Speech or Debate in either House, they shall not be questioned in any other Place. All past legislative acts are inadmissible. Promises, future acts are admissible because is not legislative. Persons Covered - The congressional committee members, members of their staff, the consultant, and the investigator are absolutely immune under the Speech or Debate Clause insofar as they engaged in the legislative acts of compiling the report, referring it to the House, or voting for its publication. Doe v. McMillian Scope of Immunity - Conduct that occurs in the regular course of the legislative process and the motivation behind the conduct is immune from prosecution. - Speech and debate clause prohibits inquiry into things done by the senators, agents, and assistants which would have been legislative acts. Therefore, its privileged. Private Publications are not protected if it is in no way essential to the deliberations of the senate. Gravel v. US - Under the Speech or Debate Clause, evidence of a legislative act of a Member of Congress may not be introduced by the Government in a

27

prosecution. Members of congress may be prosecuted under a criminal statute provided that the government does not rely on legislative acts or the motivation for legislative acts. U.S. v. Helstopski While the exclusion of evidence of past legislative acts undoubtedly will make prosecutions more difficult, nevertheless, the Speech or Debate Clause was designed to preclude prosecution of Members for legislative acts. References to legislative acts of a Member cannot be admitted without undermining the values protected by that Clause. U.S. v. Helstopski

Excluded Activity/Communications Bribes-- As to what restrictions the Clause places on the admission of evidence, the concern is with whether there is evidence of a legislative act; the protection of the Clause extends only to an act that has already been performed. A promise to deliver a speech, to vote, or to solicit other votes is not "speech or debate" within the meaning of the Clause, nor is a promise to introduce a bill at some future date a legislative act. U.S. v. Helstopski - Speeches outside of congressspeeches and publications made outside of congress is not protected. The Clause does not afford absolute immunity from private suit to persons who, with authorization from Congress, perform the function, which is not part of the legislative process, of publicly distributing materials that allegedly infringe upon the rights of individuals. Doe v. McMillian. - News Letters/Press release--The Speech or Debate Clause does not protect transmittal of information by individual Members of Congress by press releases and newsletters. Even if those acts are immune on the floor, if given in a press release or newsletter, it is no longer protected because the subsequent speech is not a legislative act because the speech was directed to the public. Hutchinson v. Proxmire. The newsletters and press release were not privileged as part of the "informing function" of Members of Congress to tell the public about their activities. Individual Members' transmittal of information about their activities by press releases and newsletters is not part of the legislative function or the deliberations that make up the legislative process; in contrast to voting and preparing committee reports, which are part of Congress' function to inform itself, newsletters and press releases are primarily means of informing those outside the legislative forum, and represent the views and will of a single Member. Hutchinson v. Proxmire - Defamatory Statements-- There is nothing in the history of the Clause or its language suggesting any intent to create an absolute privilege from liability or suit for defamatory statements made outside the legislative Chambers; precedents support the conclusion that a Member may be held liable for republishing defamatory statements originally made in the Chamber. Hutchinson v. Proxmire Waiver of Protection

28

Cannot waive the protection of the clause by testifying before the grand jury and voluntarily producing documentary evidence of legislative acts. Assuming, without deciding, that a Member of Congress may waive the Clause's protection against being prosecuted for a legislative act, only after explicit and unequivocal renunciation of the protection. U.S. v. Helstopski Congress could constitutionally waive the protection of the Clause for individual Members, such waiver could be shown only by an explicit and unequivocal legislative expression. U.S. v. Helstopski

Presidential Power
Lawmaking power is entrusted solely to Congress. The Presidents power to issue and enforce an order must stem from either an act of Congress or from the Constitution itself. Youngstown sheet & tube CoA president ordered the secretary of state to seize all steel companies (private companies). The court held that s Presidential order to seize property is invalid if it is not founded on a legislative statute or in the constitutional provisions that grant and delimit executive powers. The Constitution limits the Presidents role in the lawmaking process to recommending and vetoing legislation. The president may act over internal affairs when: o (1) The President acts with the express or implied authority from Congress (act with own powers under Art II plus powers of congress Art. I) (max) o (2) cases in which Congress had thus far been silent (twilight zone)(acts upheld as long as the act does not take over the powers of another branch) o (3) relying only on Constitutional granted power under Art. II (min)

Power over External affairs 1. Warpresident have extensive military power. The courts have dismissed as nonjudiciable political questions various challenges to the Presidents authority to take military action. 2. Foreign Relationsrepresent and act for US in day-to-day foreign relations 3. Treaty powergranted to the president with advice and consent of the senate o Self-executing treatieseffective without any implementation form congress o Non-self-executingcongress must pass legislation (not supreme until implemented, but treaty itself can be used for basis of congress to adopt required legislation) o Executive agreementsbinding agreements made by the president on the US. No authorization of congress required. The constitution does not expressly authorize

29

executive agreements, however they have been exercised and is a part of the law of the land. Executive agreements made without legislative authorization depends on the presidents possessing constitutional authority to act without congressional sanction. i.e. foreign affairs, international agreements, etc. The president may use executive agreements to accomplish foreign policy goals, even when those goals involve domestic matters Executive agreement are the not the legal equivalent over federal statutes, but a treaty if enacted later does The supreme court on many occasions have upheld the presidents power to act through executive agreements even if the president could have chosen to submit the matter to the senate to form a treaty. Goldwater v. carterDeals with the presidents power to terminate a treaty. The constitution is silent on whether the president has the authority to terminate a treaty. dispute between Congress and the President is not ready for judicial review unless and until each branch has taken action asserting its constitutional authority.... The Judicial Branch should not decide issues affecting the allocation of power between the President and Congress until the political branches reach a constitutional impasse. Political Questionnonjudiciable) Pink v. US -- No state can rewrite our foreign policy to conform to its own domestic policies. Power over external affairs is not shared by the States; it is vested in the national government exclusively." All international compacts and agreements have the same dignity and legislative supremacy as treaties and therefore must be honored by the states. Dames and Moore v. Regan(executive agreement without statutory authorization having an effect on domestic relations, president had the power to settle claims of the US against foreign government through executive agreements) Congress has implicitly authorized the President to suspend the claims of American citizens against foreign powers by its long history of acquiescing to such Presidential conduct. The Court held that the International Emergency Economic Powers Act constituted a specific congressional authorization for the President to order the transfer of Iranian assets. The Court further held that although the IEEPA itself did not authorize the presidential suspension of legal claims, previous acts of Congress had "implicitly approved" of executive control of claim settlement.

The war power resolution - Only congress can declare war. Art. 1 8 cl. 11 - Resolution provides that if the president introduces US forces into hostilities in absence of declaration of war, he must issue a report in writing to the speaker of the house and president of the senate and terminate use of force with 60 days thereafter unless congress authorized such use of force, extend the 60 day period, or is physically unable to meet

30

because of an armed attack on the US. This is a response to NATO where the president to could respond to a sudden attack under theories of self defense, etc. This issue is non-judiciable

Detention of Enemy Combatants - Rasul v. BushThe sole question before the Supreme Court in this case is whether foreign nationals detained in Guantanamo Bay may invoke habeas corpus (wrongful detainment) at all. Either U.S. citizenship or court jurisdiction is necessary for this invocation, and since the detainees are not citizens, U.S. court jurisdiction over Guantanamo Bay was at issue. According to the U.S. treaty with Cuba over Guantanamo Bay, the U.S. has "plenary and exclusive jurisdiction" over the base, but Cuba has "ultimate sovereignty." o Because the petitioners were held in federal custody in US jurisdiction in violation of the law of US, the Statute 2241 Authorized Use of Military Force, confers upon the federal courts jurisdiction to hear habeas corpus writ of foreign detainees. They do not have to be a US citizen to file writ b/c us have de facto control over Guantanamo Bay. - Hamdi v. Rumsfeld(Illegal Enemy Combatant) The Court recognized the power of the government to detain enemy combatants, but ruled that detainees who are U.S. citizens must have the ability to challenge their enemy combatant status before a neutral decision maker. They are given the basic 5th amendment due process rights. This protection is enforceable through judicial review. o Congress had expressly authorized the detention of enemy combatants in its Authorization for Use of Military Force (AUMF) passed after 9/11, due process required that Hamdi have a meaningful opportunity to challenge his enemy combatant status. o Detainee can be a US citizen and a unlawful combatant and may be locked up until the end of combat - Boumedenene v. BushNon US citizen had a right to the habeas corpus under the United States Constitution and that the MCA was an unconstitutional suspension of that right. Aliens detained as enemy combatants on that territory (Cuba) were entitled to the writ of habeas corpus protected in Article I, Section 9 Cl. 2 of the U.S. Constitution. o The majority found that the constitutionally guaranteed right of habeas corpus review applies to any person held in Guantanamo and to persons designated as enemy combatants on that territory. o If Congress intends to suspend the right, an adequate substitute must offer the prisoner a meaningful opportunity to demonstrate he is held pursuant to an erroneous application or interpretation of relevant law, and the reviewing decision-making must have some ability to correct errors, to assess the sufficiency

31

of the government's evidence, and to consider relevant exculpating evidence. The Detainee Treatment Act of 2005 failed to provide an adequate substitute for habeas corpus.

Treaties
The Constitution assigns the power to the president, with advise and consent of 2/3 of the senate the power to make treaties. Art. II 2 cl. 2. Treaties are the supreme law of the law and trumpts state laws. If treaty is in conflict with federal law, the last law enacted prevails. Treaties and State Power - Missouri v. Hollandstate challenged the constitutionality of a treaty that that regulated the shooting of migratory birds under the 10th amendment. o The constitution holds no textual limits on treaty power when a national interest is at issue. The 10th amendment does not limit treaty making power because it is enumerated in the constitution and necessary and proper clause. The states are delegated powers under the 10th amendment that are not delegated to the federal government. o The constitution does not limit congress to its enumerated powers when implementing a treaty. Under the necessary and proper clause, congress may enact all laws necessary to execute a valid treaty. In implementing a treaty, congress could pass legislation it did not have the power to act under Article 1. o Congress may not use its treaty power to enact laws that would violate a specific constitutional prohibition, such as a provision of the bill or rights. o Self-executing treatytakes immediate effect such as domestic law o Non-self-executing treatyrequires implementing by congress before it becomes enforceable. o Bricker amendmenta treaty would become effective as international law only through legislation that would be valid in absence of a treaty.never approved. - Whitney v. Robertsona treaty is on the same footing as acts of legislation. If a treaty is inconsistent with federal law, the last one to date takes control. If they are not inconsistent, they work together. o If the statute trumpts the treaty, the US is still on the hook for breach of international law and will owe damages o The US cannot enter a treaty that violates the US Constitution

Executive Privilege
There is a presumptive privilege fro presidential communications based on the presidents expectation for confidentiality of personal conversations and correspondence. The privilege is

32

only qualified. A claim of privilege based only on the general interest in confidentiality cannot prevail over the demonstrated specific need for evidence in a pending criminal trial. - Claim that disclosure would reveal national security interest would be balanced, court may be an in camera inspection. Criminal case Rejected the claim of absolute immunity in the president from judicial proceedings based on the separation of powers. The President was subject to produce information relating to a criminal prosecution. He was compelled. The court stated that the result may be different given the need to protect military, diplomatic, or national security secrets. US v. Nixon. o Presidential recording and material preservation act-- is a landmark court case concerning the principle of presidential privilege and whether the public is allowed to view a Presidents confidential documents. The Presidential Recordings and Materials Preservation Act ordered that the Administrator of General Services obtain President Richard Nixons presidential papers and tape recordings. In addition, the Act further ordered that government archivists seize these materials. These archivists would preserve the material deemed historic and return to former President Nixon the materials deemed as private. Nixon v. AGS o The act did not violate the presidents privilege of confidentiality. The review of doc. by the government archivist would be no more intrusive then the in camera proceedings permitted by the SC in Nixon. The intrusion was not a violation of privacy because it was screened and the general public has a legitimate reason to know about the presidents acts. Nixon v. AGS o Executive privilege can be claimed by former presidents. Civil Liability The president is immune from civil suits for damages caused by official acts. Has absolute Immunity. The president is not immune criminal acts stemming from his official and unofficial duties. Nixon v. Fitzgerald. o The presidents absolute immunity does not extend to his aids unless the functions they are performing demand such immunity. They have qualified immunity. Qualified immunity would not be available if the official knew or had reason to know that the actions violated the P constitutional rights or if the action was taken with malicious intent to cause a deprivation of constitutional rights (objective reasonableness test). The official claiming absolute immunity has the burden to demonstrate that the responsibilities of his office embrace such a sensitive function that requires a total shield from liability. o Immunity does not extend to suits for damages arising out of event s that occurred before the president took office or acts outside of official duties. Clinton v. Jones

33

Pardon PowerA parden is the forgiveness of a crime. Once the pardon is issued, all the punishment of the crime is waived. It may be exercised at anytime after its commission, during legal proceedings, pendacy, or after conviction. This is a presidential power and not subject to congressional control. The president may impose conditions on a pardon.

Line-Item Veto
The Supreme Court of the United States ruled that the line-item veto as granted in the Line Item Veto Act of 1996 violated the Presentment Clause of the United States Constitution because it impermissibly gave the President of the United States the power to unilaterally amend or repeal parts of statutes that had been duly passed by the United States Congress. The bill must be approved or rejected by the President in its entirety. Clinton v. NY In some situations, Congress authorizes and appropriate funds for various programs but the President impounds-refuses to send-the money. When congress has expressly directed that sums be spent, the president has no constitutional power not to spend them. If congress does not make such an explicit command to spend appropriated funds, Train appeared to agree that the President could impound the funds. If the statute gives the President discretion to spend, no case has rejected a Presidental Power.

Bill of Attainder
A bill of attainder is an act of a legislature declaring a person or group of persons guilty of some crime and punishing them without benefit of a judicial trial. The United States Constitution forbids legislative bills of attainder under Article I, Section 9. Within the U.S. Constitution, the clauses forbidding attainder laws serve two purposes. First, they reinforced the separation of powers, by forbidding the legislature to perform judicial or executive functionssince the outcome of any such acts of legislature would of necessity take the form of a bill of attainder. Second, they embody the concept of due process, which was partially reinforced by the Fifth Amendment to the Constitution. The text of the Constitution, Article I, Section 9; Clause 3 is "No Bill of Attainder or ex post facto Law shall be passed". The constitution of every State also expressly forbids bills of attainder. - Nixon v. Administrator of General Services, 433 U.S. 425 (1977). In 1974, Congress passed the Presidential Recordings and Materials Preservation Act, which required the General Services Administration to confiscate former President Richard Nixon's presidential papers to prevent their destruction, screen out those which contained national security and other issues which might prevent their publication, and release the remainder of the papers to the public as fast as possible. The Supreme Court upheld the law in Nixon, arguing that specificity alone did not invalidate the act because President constituted a "class of one." Thus, specificity was constitutional if it was rationally related to the class identified. The Court modified its punishment test, concluding that

34

only those laws which historically offended the bill of attainder clause were invalid. The Court also found it significant that Nixon was compensated for the loss of his papers, which alleviated the punishment. The Court further modified the punishment prong by holding that punishment could survive scrutiny if it was rationally related to other, nonpunitive goals. Finally, the Court concluded that the legislation must be intended to punish; legislation enacted for otherwise legitimate purposes could be saved so long as punishment was a side-effect rather than the main purpose of the law.

Appointment and Removal Power


The constitution allows congress to create offices and the right to appoint to them, this is done through the necessary and proper clause. Yet, congress lacks the power to appoint persons to fill the offices it creates. (checks and balances). - The president appoints, sometimes with the advice and consent of the senate - Principle officers are appointed by the president only with the advice and consent of the senate (ambassadors, public ministers, judges of the SC, officer of the US, federal officers) - Inferior officers do not require senate confirmation and may be appointed by someone other than the president. (congress can appoint or decide who appoints them, appointment powers of some inferior can be vested in the judiciary) o Morrison v. Olson(Appointment of Independent Counsel/Special Prosecutor by Congress is permitted). The Ethics in Government Act of 1978 created a special court and empowered the Attorney General to recommend to that court the appointment of an "independent counsel" to investigate, and, if necessary, prosecute government officials for certain violations of federal criminal laws. The act grants the counsel full power to authority to investigate procedural functions. IC can only be removed by impeachment and convention, or by Attorney general for good cause or if conditions substantially impair the performance of duties. The court held that the independent counsel was an inferior officer because (1) he was subject to removal by a higher executive official (attorney general) (2) empowered to perform limited duties (3) had limited JD (4) limited tenure. The Court upheld the Independent Counsel provision because it did not violate the separation of powers by increasing the power of one branch at the expense of another. Instead, even though the President could not directly fire the independent counsel, the person holding that office was still an Executive branch officer, not under the control of either U.S. Congress or the courts (the AG). Removal - Article 2 tells us how people may be appointed but not how the position may be terminated. - By Presidentthe president can remove high level, purely executive officers (cabinet members) at will without interference from Congress. The president does not have

35

absolute power to remove members of an agency that operates free from executive interference. However, congress may provide statutory limitations on the presidents power to remove independent agencies created by congress. Even then, congress cannot eliminate all executive removal power (removal for good cause). By Congresscongress cannot give itself power to remove an officer charged with the execution of laws except through impeachment. A congressional attempt through legislation to remove from the government employment specially named government employees is likely to be an invalid bill of attainder. o Allowed the Comptroller General to report, make recommendations on budget reduction to the president in an effort to control the federal deficit. This act was challenged on the grounds that it imposed executive functions on the Comptroller General, who was not a member of the executive branch. The power of Congress to remove the Comptroller General was an important factor in the Courts conclusion that the Constitution was violated. Reserving for itself the power of removal of an officer charged with execution of the laws, congress gained control over executive functions forbidden to it by the Constitution. The act was unconstitutional because it gave executive power to an officer who could be removed by congress. (Bowsher v. Synar) Morrison v. Olson--the president can fire the Attorney General for any reason because the attorney general is a member of the executive branch. However, the president has to show good cause to remove the independent counsel. The good cause provision does not burden the presidents power to control or supervise the independent counsel, as an executive official, in the execution of her duties. The attorney general can remove the independent counsel for good cause. Congress may limit the presidents freedom to dismiss certain executive officials so long as such restrictions do not compromise the presidents ability to fulfill its constitutional duties.

Term Limits on US Senators and Representatives


was a case in which the Supreme Court of the United States ruled that states cannot impose qualifications for prospective members of the U.S. Congress stricter than those specified in the Constitution. - Qualifications clause Art 1.Sect. 2age residency, citizenship is the qualification. It is exclusive and a state may not expand it - Election clausethe times place and manner of holding elections for senators and representatives shall be proscribed by each state by the legislature - Cook v, Gralike--In the wake of U.S Term Limits v. Thornton, Missouri voters adopted an amendment to Article VIII of their State Constitution, which "instructs" each Member of Missouri's congressional delegation "to use all of his or her delegated powers to pass the Congressional Term Limits Amendment" set forth in section 16 of the Article. The

36

Article also directs the Missouri Secretary of State to determine whether a statement reflecting a candidate's position on term limits should be placed by his or her name on the general election ballot. It was an unconstitutional attempt to regulate electoral outcomes, rather than a permissible regulation of state elections. It was designed to favor candidates who are willing to support a term-limits amendment and, "[t]hus, far from regulating the procedural mechanisms of elections, and attempts to 'dictate electoral outcomes.' Such 'regulation' of congressional elections simply is not authorized by the Elections Clause." Add supplement case pg. 9

Ch. 6 Due Process Clause


14th Amendment Section 1. All persons born or naturalized in the United States, and subject to the jurisdiction thereof, are citizens of the United States and of the State wherein they reside. No State shall make or enforce any law which shall abridge the privileges or immunities of citizens of the United States; nor shall any State deprive any person of life, liberty, or property, without due process of law; nor deny to any person within its jurisdiction the equal protection of the laws.

14th amendment privileges and Immunities


Prohibits states from denying their citizens the P&I of national citizenship, such as right to vote for federal offices, right o travel, right to DP. 1. Corporations are not protected/ only protects citizens 2. Bill of rights not includedSlaughterhouse cases, held that fundamental rights protected against federal abuse (first 10 amendments) are not privilege and immunities of national citizenship within the meanings of the 14th amendment; nor is right to work, live, eat. Thus the guarantee of the bill of rights are protected from state action only by DP and EP of the 4th amendment 3. Right to travel--protected by the 14th amendment, included the rights of newly arrived citizens to enjoy the same privileges and immunities are enjoyed by other citizens of the state. o Saenz v. Roe--it has always been common ground that this Clause protects the third component of the right to travel. Writing for the majority in the SlaughterHouse Cases, one of the privileges conferred by this Clause "is that a citizen of the United States can, of his own volition, become a citizen of any State of the Union by a bona fide residence therein, with the same rights as other citizens of that State." 14th amendment only protected P&I of citizens of the United States, thus only prohibited interference with certain privileges of national citizenship. (protection of fundamental

37

property rights and civil rights was within the JD of the state, not the federal government). o Federal privileges and immunities were limited to those rights which owe their existence to the federal government or constitution. the second sentence forbidding states from making "any law which shall abridge," only applied to federal rights alone. Slaughter-House CasesNew Orleans passed a law granting exclusive monopoly rights to slaughter animals in the New Orleans area to the Slaughter-House Company. The Plaintiff claim that the law prohibited them from practicing their trade and it deprived them from their property. Court held that it was not a violation of the privileges and immunities clause because it was not incorporated to the states P&I of US citizenship only protected national/federal rights. Challenges raised 1. It created voluntary involuntary servitude prohibited by the 13th amendment (limited to slavery, court struck it down) 2. It abridged the privilege and immunities of the 14th amendment 3. It denied the butches equal protection of the law 4. It deprived the butchers of their property without due process of the law (all were rejected)

Incorporation The federal constitution prohibited the states from denying its citizens full protection under the Bill of Rights but did not incorporate it to the states per se. The due process/fundamental fairness notion requires that a states judicial proceedings be fundamentally fair in all respects. Only fundamental rights are incorporated to the states.

Procedural Due Process


The requirements of procedural due process only applies to the deprivation of interests encompassed by the 14th amendments protection of liberty and property. When protected interest are implicated, the right of some kind of prior hearing is paramound. To determine the form of hearing required, a weighing process is used. When is DP Required The right to procedural DP is required when the gov. acts to deprive an individual of life, liberty, property. Is Life, Liberty, or Property being Taken The court must determine whether a legitimate business interest is being taken. To determine whether due process requirements apply, must look at the interest at stake (it must be within the 14th amendment protection of liberty and property). To have a property interest, the person must have a legitimate claim of entitlement to it. Property interest is not created by the constitution.

38

They are created and defined by existing rules or understandings that stems from an independent source of state law. Liberty 1. Liberty can mean freedom of bodily restraint, right to contract, engage in gainful employment. A deprivation of liberty occurs if a person o Loses significant freedom of action o Is denied a freedom provided by the constitution or statute Property - Welfare benefitsone has property interest in welfare benefits if it has been determined that she meets statutory criteria - Continued Public/Gov employmentstatute or ordinance that creates public employment contract and employee can only be terminates for good cause, then there is a property interest. (no property interest for at will employment. - The lack of contractual or tenure right to reemployment alone does not defeat a claim that the nonrenewal of a contract violated the 14th amendment. A persons interest in a benefit is a property interest for due process purposes if there are such rules or mutually explicit understandings that supports his claim of entitlement to the benefit and that he may invoke at a hearing. o Board of Regents v. RothDavid Roth was hired as a first year assistant professor of political science in 1968 for a fixed term of one year. In accordance with procedural rules set by the Board of Regents, the president of the University informed Roth he would not be rehired for the next academic year, giving him no reason for the decision and no opportunity to challenge it in any sort of hearing. He alleged violation of 14th amendment substantive and procedure due process. The Supreme Court held that the Fourteenth Amendment does not require an opportunity for a hearing prior to the nonrenewal of a non-tenured state teacher's contract because the nonrenewal did not deprive him of "liberty" or that he had a "property" interest in continued employment. o Perry v. Sindermanna non-tenure teacher who has held his position for a significant number of years might be able to show that he has a legitimate claim of entitlement to job tenure. The respondent has alleged existing rules of understanding that may justify his legitimate claim of entitlement so he should be afforded a hearing. Proof of property interest or granting of a hearing to challenge sufficiency does not entitle one to reinstatement. o DeShaney v. Winnebagoa four year old boy was beaten to death by his father over a period of time placing him in a coma, the social workers had received official complaints but did not remove him. The petitioner claimed that the failure to act deprived him of liberty interest in DP of the 14 amendment. The court held

39

that nothing in the language of the DC itself requires a states to protect the life, liberty, and property of its citizens against private actors. The clause is limited on the states power to act. o Castle Rock v. GonzalezWoman received a restraining order against her husband and he violated it. She called the police, they failed to enforce it. The court held that a person who has obtained a restraining order does not have a constitutionally protected property interest in having the police enforce it. Determining what process is due All intentional gov. deprivations of life liberty, or property require fair process. To determine what process is due the court weighs 1. Importance of the individual interest involved 2. The value of specific procedural safeguards to that interest 3. The government interest in fiscal and administrative efficiency Termination of Government benefits/Per-Termination hearingthe legislation can confer property interest in employment/benefits, etc but once property interest are created, they cannot be constitutionally be taken away without appropriate procedural safeguards. - Welfare Benefits--Goldberg v. Kellythe Due Process Clause of the Fourteenth Amendment to the United States Constitution requires an evidentiary hearing before a recipient of certain government benefits (welfare) can be deprived of such benefits. The individual losing benefits is not entitled to a trial, but is entitled to an oral evidentiary hearing before an impartial decision-maker before the termination of benefits, the right to confront and cross-examine witnesses, and the right to a written opinion setting out the evidence relied upon and the legal basis for the decision. Welfare benefits are a matter of statutory entitlement for persons qualified to receive them and procedural due process is applicable to their termination. The interest of the eligible recipient in the uninterrupted receipt of public assistance, which provides him with essential food, clothing, housing, and medical care, coupled with the State's interest that his payments not be erroneously terminated, clearly outweighs the State's competing concern to prevent any increase in its fiscal and administrative burdens. - Disability benefits--Mathews v. Elderidgethat individuals have a statutorily granted property right in social security benefits, that the termination of those benefits implicates due process, but that the termination of Social Security benefits does not require a pretermination hearing because the present administrative procedures fully comfort with DP. No prior evidentiary hearing required as long as prior notice was given to the recipient, an opportunity to respond was given, and a subsequent evidentiary hearing was provided. Disability benefits are unlike welfare benefits, not based on financial needs and are not as vital.

40

The essential requirements of due process are notice and the opportunity to respond. The opportunity to present reasons why a proposed action should not be taken is a fundamental requirement of due process.

Cleveland v. Board of Regents Ohio employees, both "classified civil servants" under Ohio law, were terminated without being afforded a pre-termination hearing to respond to the charges. A security guard (James Loudermill) employed by the Cleveland Board of Education was dismissed for failing to disclose a prior felony conviction for grand larceny on his job application. The Ohio law stated that "classified civil servants" could be terminated only for cause and were entitled to post-termination administrative review of the decisions. The Ohio statute providing for administrative review, was unconstitutional on its face because it provided no opportunity for a discharged employee to respond to charges against him prior to removal, thus depriving him of liberty and property without due process. 1. Public employees subject to removal only for cause must be given notice of charges that are basis for termination 2. A pre-termination opportunity to respond 3. Subsequent evidentiary hearing regarding the termination, Notes: If there is a significant reason for not keeping employee on the job, he may be suspended without pay and without an opportunity to respond as long as there is a prompt post-suspension hearing without an opportunity to respond as long as there is a prompt post-suspension hearing with reinstatement and back pay if the employee prevails Public EducationDisciplineno formal evidentry hearing require before a student may be temporarily suspended , due process usually requires notice of the charges and opportunity to explain. If student poses a danger, notice and hearing may follow removal as soon as practicable Public EducationAcademic dismissalno prior hearing is required when a student is dismissed for academic deficiencies rather than for disciplinary reasons. DP is satisfied if the student is adequately informed of the deficiencies and given opportunity to respond. Creditors remediesbefore wage garnishment, must have notice and hearing prior to issuance of order Drivers license suspendedprior hearing required before suspension. Post-suspension hearing satisfies due process where a statute mandates suspension of a drivers license for refusing to take a breathalyzer test upon arrest for drinking and driving

Economic Penalties Punitive Damages When there is no procedure for reducing excessive punitive damage awards that are excessive, it is a violation of due process. Grossly excessive punitive damage awards can be a violation of due process because it is a arbitrary deprivation of property, requires due process. 1. Degree of reprehensibility of the Defendants conduct

41

2. The ratio between the punitive damages and actual harm 3. And the difference between the punitive award and civil or criminal sanctions imposed for comparable misconduct.

Economic Regulation and the contract clause Article 1 Section 10


Contract Clause Article 1 10 Prohibits states, but not the federal government from passing a law impairing the obligation of contracts. 1. The contract clause is not applicable to the federal government. However if brought against the federal government it would analyzed under DP of 5th amendment. 2. Only applies to state legislation 3. Basic impairment rules o Private Contractsonly prevent substantial impairments of contracts. Must ask (1) does the legislation substantially impair a partys rights under an existing contract? If no, the legislation is valid, if yes, only valid if it serves an important and legitimate public interest and is a reasonable and narrowly tailored means of promoting that interest. o Public contractsthose in which the state or political subdivision is a party tested by same test, but may receive stricter scrutiny (1) there is no substantial impairment if the state reserves the power to revoke, alter, or amend the contract itself of terms in the contract (2) in determining whether the law serves a legitimate public interest, the state cannot be obliged by the contract to refrain from exercising its police power necessary to protect the health and safety of residents, and (3)to be narrowly tailored, it cannot be unnecessarily broad repudiation of the contract obligations.

The contract clause prevents only substantial impairment of contracts. - Test: the state law must substantially impair the contractual relationship, the state must justify its action by having a significant and legitimate public purpose behind the regulation, such as the remedy of a broad and general social or economic problem. The adjustment of the rights and responsibilities of the parties must be reasonable and appropriate. - Home Building & Loan Association v. Blaisdell, 290 U.S. 398 (1934), was a decision of the United States Supreme Court holding that Minnesota's suspension of creditors' remedies was not in violation of the United States Constitution. Blaisdell was decided during the height of the Great Depression .In 1933, in response to a large number of home foreclosures, Minnesota, like many other states at the time, extended the time available for mortgagors to redeem their mortgages from foreclosure. The extension had the effect of enlarging the mortgagor's estate contrary to the terms of the contract. The

42

Supreme Court upheld the statute, reasoning that the emergency conditions created by the Great Depression "may justify the exercise of [the State's] continuing and dominant protective power notwithstanding interference with contracts."[5] Blaisdell was the first time the court extended the emergency exception to purely economic emergencies. Allied Structural Steel Co. v. Spannus--In 1974, Minnesota adopted legislation which required private employers to pay a fee if they terminated employee pension plans or if they moved their offices from the state, leaving insufficient funds to cover pensions for ten-year employees. This law affected Allied Structural Steel as the company began closing offices in Minnesota. Even though the employees affected by the closing were not entitled to pensions under the terms of their employment with the company, according to the Minnesota law, they were. The company was ordered to pay approximately $185,000 to comply with the statute's provisions. The Court found that the Minnesota law did violate the Constitution as it "substantially altered" the provisions of pension agreements which Allied Steel had with its employees. o It did not deal with a broad, generalized economic social problem. o Invaded an area never before subject to regulation by the state o The regulation did not temporary alter the contractual relationships, it was permanent United States Trust Co v. New Jerseypolice power an imminent domain were powers that could not be contracted away, but the state can bind itself in the future exercise of its taxing and spending powers. Contract clause only applies to the state. When its the federal government, must bring a DP claim.

Government taking of private property


The Fifth Amendment prohibits the government from taking private property, for public use without just compensation. Taking Includes physical appropriation, government action that significantly damages or impairs its use. - United States v. Causby-- In the case, Causby sued the United States for trespassing on his land, complaining specifically about how "low-flying military planes caused the plaintiffs' chickens to 'jump up against the side of the chicken house and the walls and burst themselves open and die' . . . The plaintiffs sued the government, arguing that they were entitled to compensation under the takings clause of the Fifth Amendment. The court held that if the landowner is to have full enjoyment of the land, he must have exclusive control of the immediate reaches of the enveloping atmosphere. Otherwise buildings could not be erected, trees could not be planted, and even fences could not be run" . . . Thus, a landowner "owns at least as much of the space above the ground as he

43

can occupy or use in connection with the land," and invasions of that airspace "are in the same category as invasions of the surface. Military Actionsgovernmental occupation of private property deprives the owner of his use of the property and it is deprivation in which the constitution requires compensation. No compensation when the use of the building by military troops did not deprive the owner of the use of property because the building was already under attack so building could not be used by petitioner anyway. No direct and substantial invasion to require just compensation. No violation of due process of taking clause when an owners interest in property is forfeited by reason to the use at which the property is put even if the owner did not know that the property was put to such use. Kaiser v. Aetna v. United Statesprivate marina was open only to fee paying member, excluded navigation by the public. (it was linked to navigable waters). The right to exclude is so universal to be held a fundamental element of property rights, falls within the category of interest that the government cannot take without just compensation. The imposition of navigatable servitude in this context will result in an actual physical invasion of a privately owned marina. Pruneyard Shopping Center v. Robinsplace open to the public at large sought to exclude petitioning. Court held that shopping center was not unreasonable impaired, did not interfere with its commercial function, it was open to the public, and limited their activity to the common areas of the shopping center. The fact that they physically invaded the property is not determinable. Nollan v. Californai Coastal Commissionit was a taking when the California coastal commission granted a permit to Petitoner to replace their beach front bungalow with a larger house on the condition that they allow the public an easement to pass across the Nollans beach. It violated the taking clause because it did not further some government purpose, not proper exercise of the use of land regulations. (general rule, cant condition issuing a permit on deeding land out to the government. Must be a nexus between legitimate state interest and the permit.

Public Use - Kelo v. New London--was a case decided by the Supreme Court of the United States involving the use of eminent domain to transfer land from one private owner to another to further economic development. The case arose from the condemnation by New London, Connecticut, of privately owned real property so that it could be used as part of a comprehensive redevelopment plan which promised 3,169 new jobs and $1.2 million a year in tax revenues. The Court held in a 54 decision that the general benefits a community enjoyed from economic growth qualified such redevelopment plans as a permissible "public use" under the Takings Clause of the Fifth Amendment.

44

Taking By regulation The government does not need to pay for regulation of property use. However, government regulation can amount to taking. A taking by regulation has occurred when: 1. The extent to which the regulation has interfered with a distinct investment back expectation. 2. The character of the governments action (i.e. physical invasion) 3. Economic impact of the gov. actions

Village of Belle Terre v. Boraas--United States Supreme Court case in which the Court upheld the constitutionality of a residential zoning ordinance that limited the number of unrelated individuals who may inhabit a dwelling.The Supreme Court held that the Belle Terre ordinance was a constitutional restriction on the use of land. It further held that the police power is a valid basis for establishing residential zones limiting the number of unrelated individuals that may inhabit a dwelling. - There was no protected class discrimination, and no fundamental right infringed by the ordinance, the proper standard of review was the rational basis test. - The legislature should define a family, rather than the judiciary. As long as there exists a rational basis for the legislature's determination, it would be upheld by the courts. - The ordinance did not restrict the freedom of association, as homeowners may entertain whomever it wants and that the ordinance was only a restriction on who could permanently live in that residence. - The city had a rational basis for its prohibition on housing large numbers of unrelated individuals because creating a quiet neighborhood is a permissible state goal and this ordinance is closely related enough to this goal to be sustained under the rational basis test. Moore v. City of East Cleveland--The Court held 5-4 that an ordinance which restricted housing to a single family and defined the family as a nuclear family, rather than an extended family, was unconstitutional and a violation of the Due Process Clause of the Fourteenth Amendment. This case was distinguishable from Village of Belle Terre v. Boraas, because that ordinance only affected unrelated individuals. - The ordinance here limits its definition of family to the nuclear family, a relatively new conception. - When the government intrudes on choices concerning family living arrangements, the usual deference to the legislature is inappropriate, and the Court must examine carefully the importance of the governmental interests advanced and the extent to which they are served by the challenged regulation.

45

The ordinance had a weak relationship to the objectives cited by the city such as avoiding overcrowding, traffic congestion, and an undue financial burden on the school system because a nuclear family could still have a much larger impact on these than a small group of extended family living together.

Regulations of Business and other property interest Substantive Due Process Is one of the theories of law through which courts enforce limits on legislative and executive powers and authority. The avenues for use of this theory by courts are the Due Process Clauses of the Fifth and Fourteenth Amendments, which prohibit the federal and state governments, respectively, from depriving any person of "life, liberty, or property, without due process of law. SDP aims to protect individuals against policy enactments which exceed the limits of governmental authoritythat is, courts find the majority's enactment is not law, and cannot be enforced as such, regardless of how fair the process of enforcement actually is. The Court usually looks first to see if there is a fundamental right, by examining if the right can be found deeply rooted in American history and traditions. Where the right is not a fundamental right, the court applies a o rational basis test: if the violation of the right can be rationally related to a legitimate government purpose, then the law is held valid. o If the court establishes that the right being violated is a fundamental right, it applies strict scrutiny. This test inquires into whether there is a compelling state interest being furthered by the violation of the right, and whether the law in question is narrowly tailored to address the state interest.

Substantive Due Process


Economic Regulation - The Fourteenth Amendment protected an individual's "general right to make a contract in relation to his business." He acknowledged that the right was not absolute, referring to the "somewhat vaguely termed police powers" of the state. The police power was subject to certain limitations; otherwise, he claimed, the Fourteenth Amendment would be meaningless, and states would be able to pass any law using the police power as a pretext. He asserted that it was the court's duty to determine whether legislation is "a fair, reasonable and appropriate exercise of the police power of the State, or ... an unreasonable, unnecessary and arbitrary interference with the right of the individual ... Liberty to Contractthe freedom of contract is protected as a liberty and property right under the DP clause of the 14th amendment and that right could not be regulated by the government without a legitimate gov. purpose. o Lochner v. NY-- "liberty of contract" was implicit in the due process clause of the Fourteenth Amendment. A New York law that limited the number of hours that a

46

baker could work each day to ten, and limited the number of hours that a baker could work each week to 60. The Supreme Court rejected the argument that the law was necessary to protect the health of bakers, deciding it was a labor law attempting to regulate the terms of employment, and calling it an "unreasonable, unnecessary and arbitrary interference with the right and liberty of the individual to contract." (the state must exercise its police powers within the boundaries of the 14th amendment) Neither the Fifth nor the Fourteenth Amendments prohibit governmental regulation for the public welfare; instead, they only direct the process by which such regulation occurs. As the Court has held in the past, such due process demands only that the law shall not be unreasonable, arbitrary, or capricious, and that the means selected shall have a real and substantial relation to the object sought to be attained.[ o Nebbia v. NYcourt upheld a statute that set milk prices at a set price in an attempt to save its failing milk industry by regulating prices. The court held that it was a valid exercise of the states police powers to protect the public welfare. When a state passes a law which are seen to have a reasonable relation to a proper legislative purpose, and are neither arbritrary or discriminatory, the requirements of due process are satisfied. o Ferguson v. Skrupa-- A Kansas statute[1] makes it a misdemeanor for any person to engage "in the business of debt adjusting" except as an incident to the lawful practice of law. The court held that the statute did not violate the due process clause.1) States had the power to legislate against what were found to be injurious practices in their internal commercial and business affairs so long as their laws did not run afoul of some specific federal constitutional prohibition or of some valid federal law. When the subject lay within the State's police power, debatable questions as to reasonableness were not for the courts but for the legislature.2) The Court further held that the statute's exception of lawyers did not constitute a denial of equal protection of the laws to nonlawyers. Statutes created many classifications that did not deny equal protection; it was only invidious discrimination that offended the Constitution.

47

Second Amendment District of Columbia v. Heller, 554 U.S. 570 (2008), was a landmark case in which the Supreme Court of the United States held that the Second Amendment to the United States Constitution protects an individual's right to possess a firearm for traditionally lawful purposes in federal enclaves, such as self-defense within the home. The Second Amendment protects an individual right to possess a firearm unconnected with service in a militia, and to use that arm for traditionally lawful purposes, such as self-defense within the home. Pp. 253. Like most rights, the Second Amendment right is not unlimited. It is not a right to keep and carry any weapon whatsoever in any manner whatsoever and for whatever purpose: For example, concealed weapons prohibitions have been upheld under the Amendment or state analogues. The Courts opinion should not be taken to cast doubt on longstanding prohibitions on the possession of firearms by felons and the mentally ill, or laws forbidding the carrying of firearms in sensitive places such as schools and government buildings, or laws imposing conditions and qualifications on the commercial sale of arms. Millers holding that the sorts of weapons protected are those in common use at the time finds support in the historical tradition of prohibiting the carrying of dangerous and unusual weapons. Pp. 5456. The handgun ban and the trigger-lock requirement (as applied to self-defense) violate the Second Amendment. The Districts total ban on handgun possession in the home amounts to a prohibition on an entire class of arms that Americans overwhelmingly choose for the lawful purpose of self-defense. Under any of the standards of scrutiny the Court has applied to enumerated constitutional rights, this prohibitionin the place where the importance of the lawful defense of self, family, and property is most acutewould fail constitutional muster. Similarly, the requirement that any lawful firearm in the home be disassembled or bound by a trigger lock makes it impossible for citizens to use arms for the core lawful purpose of self-defense and is hence unconstitutional. Because Heller conceded at oral argument that the D. C. licensing law is permissible if it is not enforced arbitrarily and capriciously, the Court assumes that a license will satisfy his prayer for relief and does not address the licensing requirement. Assuming he is not disqualified from exercising Second Amendment rights, the District must permit Heller to register his handgun and must issue him a license to carry it in the home.

Equal Protection

48

The equal protection clause of the 14th amendment was designed to impose upon the states a duty to prohibit legislative classifications and administrative behavior that discriminated against particular groups in the distribution of fundamental rights. - Prohibits government discrimination with respect to certain fundamental rights like voting, travel, access to the judicial process. - Prohibits discrimination against groups of people based on race, national origin, gender, illegitimacy. - EP violations standard--Requires a finding of discriminatory intent, simply proving a discriminatory effect or impact is not sufficient. Difference between EP and DP - DP is used when there is no classification. Under a due process analysis, the courts get the last say - Under a EP analysis, the legislatives get the last say, they may redraw the statute.

The Supreme Court has defined these levels of scrutiny in the following way: Strict scrutiny If the law categorizes on the basis of race or national origin or infringes a fundamental right the law is unconstitutional unless it is "narrowly tailored" to serve a "compelling" government interest. In addition, there cannot be a "less restrictive" alternative available to achieve that compelling interest. (most gov. actions fails) (Burden of Proof is on the government to prove that law is necessary) 1. Right to travel 2. Privacy 3. Voting 4. All first amendment rights Intermediate scrutiny If the law classifies based on sex or legitimacy, the law is unconstitutional unless it is "substantially related" to an "important" government interest. (BOP on gov) Rational Basis Test A law would be upheld if it is rationally related to a legitimate state interest. (most actions upheld unless arbitrary or irrational) Because the laws are presumed valid, the BOP is on the challenger 1. Business and Labor relations 2. Taxation 3. Social and Economic regulations Constitutional Source

49

EP clause of the 14th amendment is not applicable to the federal gov, it is limited to state action. However, discrimination by the federal gov. can violate DP (Bolling v. Sharpe). Test to determine the constitutionality of laws challenged under traditional, economic protection: 1. EP does not take away from the state the power to classify in the adoption of police power, its unconstitutional only when it without reasonable basis and is purely arbitrary. 2. A classification having some reasonable basis does not violate the clause merely because its not made with mathematical precision or because its practice results in some discrimination. 3. Person who assails the classification must carry the burden of showing that its not based on reasonable basis but is purely arbitrary. o Railway Express Agency v. NY--A New York City traffic regulation forbids the operation of any advertising vehicle on the streets, but excepts vehicles which have upon them business notices or advertisements of the products of the owner and which are not used merely or mainly for advertising. An express company, which sold space on the exterior sides of its trucks for advertising and which operated such trucks on the streets, was convicted and fined for violating the ordinance. EP analysisthe regulation drew the line between advisement of products sold by the owner of the trucks, and advisements for someone else Rational, the advertising constitutes a distraction to vehicle drivers and pedestrians, the court held that the classification has a relationship to its purpose and does not violate the EP. o Morey v. Doudthe community currency exchange act required that any firm selling or issuing money orders in the state must secure a license and submit to state regulations. The statute exempted American Express company money orders. The court invalidated the law based on EP because the purpose of the act was to afford public protection but the discrimination in favor of AMEX does not conform to that purpose. Metropolitan Life Ins. V. Ward.used EP clause to invalidate a statute that taxed out of state insurance companies at a higher rate than domestic insurance companies. o EP analysisPurpose, to promote the business of its domestic insurers by penalizing foreign insurers who want to do business in the state. Court stated that the purpose was purely discriminatory designed only to favor domestic industry. EP was designed to prevent this sort of discrimination.

Suspect class Invidious discriminationviolation of EP A law that classifies on its face is unconstitutional.

50

Strauder v. West Virginia--At the time, West Virginia excluded African-Americans from juries. Strauder was a Black man who, at trial, had been convicted of murder by an all-white jury. Strauder appealed his conviction, contending that West Virginia exclusionary policy violated the Equal Protection Clause of the Fourteenth Amendment. - held that categorical exclusion of blacks from juries for no other reason than their race did violate the Equal Protection Clause, since the very purpose of the Clause was "to assure to the colored race the enjoyment of all the civil rights that under the law are enjoyed by white persons, and to give to that race the protection of the general government, in that enjoyment, whenever it should be denied by the States." The Court did not say that exclusion of blacks from juries violated the rights of potential jury members; rather, such exclusion violated the rights of black criminal defendants, since juries would be "drawn from a panel from which the State has expressly excluded every man of [a defendant's] race." - The Court did not hold that any particular jury must be racially balanced in order to satisfy equal protection; the categorical exclusion from all juries was the problem. This holding is reaffirmed in the important 20th-century equal protection case Washington v. Davis: "[Strauder] established that the exclusion" of African-Americans from juries violates equal protection, but if a particular jury or series of juries "does not statistically reflect the racial composition of the community does not in itself make out an invidious discrimination forbidden by the EP clause. - Swain v. Alabamaa black D could make out a prima facie case of purposeful discrimination by proving that the preemptory challenge system as a whole was being perverted. First, he must show that he is a member of a cognizable racial group, and that the prosecutor exercised his peremptory challenges to remove from the panel members of the defendants race. The EP clause forbids the states to strike black defendants on the assumption that they will be biased in a particular case because the defendant is black. - Powers v. Ohiothe criminal defendant has suffered cognizable injury and has standing to raise a third-party equal protection claims of the jurors. Under the EP clause a criminal defendant may object to race based exclusions of jurors effectuated through preemptory challenges whether or not the defendant and the jurors are the same race. o White defendant on trial for murder, objected to the states use of peremptory challenges to remove seven black venire persons from the jury. - JEB v. Alabamasex, like race, is an unconstitutional proxy for juror proxy for competence. Cannot use preemptory challenges to remove all male, female jurors. A party alleging sex discrimination must make a prima facie showing of intentional discrimination before the party exercising the challenge must explain the basis for the strike. o The state used 9 of its 10 preemptory strikes to remove male jurors in a child support case.

51

Facially race neutral-discriminatory intent A law that is race-neutral on its face, but is administered in a prejudicial manner, is an infringement of the Equal Protection Clause in the Fourteenth Amendment if the persons challenging the law can prove that the gov. officials applying the law had a discriminatory purpose the law will be invalid. - Yick Wo v. Hopkinscourt struck down a law regulating laundries because as it was applied and administered disadvantaged only Chinese laundry owners. The law prohibited the operation of laundries in wooden building without permission. Petitioner passed heath and safety inspection but was denied a permit. Record showed that 200 Chinese were denied a permit, while 80 non-Chinese where allowed to operate such laundries. The overwhelming disproportionate impact alone was enough evidence to show discriminatory intent. Separate But Equal State - Plessy v. Fergusonthe court upheld the separate but equal doctrine against a challenge under the equal protection clause. Plessy challenged the constitutionality of a Louisiana Act requiring equal but separate railway carriages for members of different races. The court held that the EP clause only sought to to enforce political unequality, not social. - Brown v. Board of Education--was a landmark decision of the United States Supreme Court that declared state laws establishing separate public schools for black and white students unconstitutional. The decision overturned the Plessy v. Ferguson decision of 1896 which allowed state-sponsored segregation. The suit called for the school district to reverse its policy of racial segregation. Separate elementary schools were operated by the Topeka Board of Education under an 1879 Kansas law, which permitted (but did not require) districts to maintain separate elementary school facilities for black and white students in twelve communities with populations over 15,000. o The key holding even if segregated black and white schools were of equal quality in facilities and teachers, segregation by itself was harmful to black students and unconstitutional. They found that a significant psychological and social disadvantage was given to black children from the nature of segregation itself. The doctrine of separate was unconstitutional. We conclude that, in the field of public education, the doctrine of "separate but equal" has no place. Separate educational facilities are inherently unequal. Violation of EP - Browns enforcementthe court determined that the proper implementation of desegregation remedies rested with the good faith efforts of local school authorities. The school authorites must make prompt and reasonable start. The school board has the burden of showing necessity of more time. Federal Reverse incorporation

52

Applied Brown holding to the federal government through the due process clause of the 5th amendment. The 5th amendment contains no EP clause, and the 14th amendment applies only to states. The courts application of the EP clause to the federal government through the DP clause is reverse incroporation. - Bolling v. Sharpeheld that segregated schools in the District of Columbia violated the 5th amendment due process clause. - Notethe court upheld racial classifications involving curfews and relocations imposed on Japanese-Americans shortly after the Japanese attacked pearl harbor. Relying on war powers. De Facto/De Jure Segregation - Under de jure segregation, the law provided entirely separate schools for black and white students, which they legally had to attend, despite in many cases actually living closer to a school designated for the other race. - De facto segregation is segregation by fact, zoning based on neighborhood, where you live, etc. - Where there is no dual statutory system of segregation, the plaintiff must show not only that de-facto segregation existed but that it was brought about or maintain by intentional state action. Discriminatory intent is a general requirement. Once de jure segregation has affected a system, broad system wide relief is justified. If it only affects a limited part of the school district, then that part be desegregated. o Keyes v. School Dist. No. 1the school was never statutorily segregated. The plaintiff alleged the by use of various techniques such as manipulation of student attendance zones, school site selection, and neighborhood school policy, the school board created and maintained racially or ethnically segregated schools throughout the school district. - RemediesMilliken v. Bradleyrefused to find interdistrict remedy when the lower court only found only the city district of Detriot to have intentionally discriminated. First court to determine that a court order to remedy de jure segregation required more than one school district. o Missouri v. Jenkinsoverturned a District Court ruling that required the state to correct de facto racial inequality in schools by funding salary increases and remedial education programs. Racial Discrimination in Employment A gov. action appears neutral on its face, and application but have a disproportionate impact of a particular class of persons (race minority or women). To establishing an EP violation in employment requires a finding of discriminatory intent. Such intent can be proven by total exclusion of minorities or seriously disproportionate impact.

53

Washington v. Davis--The important general rule created by Washington v. Davis is that under the Equal Protection Clause of the Fourteenth Amendment, "[an] official action will not be held unconstitutional solely because it results in a racially disproportionate impact." Instead, a plaintiff must prove discriminatory intent or purpose. The court noted that "disproportionate. o While it may have been true that Test 21 had the effect of removing a greater number of black than white applicants, the test did not have a discriminatory purpose impact is not irrelevant, but it is not the sole touchstone of an invidious racial discrimination forbidden by the Constitution".

Housing and Zoning Impact was seldom sufficient standing alone to establish discriminatory intent. Once racial discrimination is shown to have been a substantial or motivating factor behind the enactment of the law, the burdern shifts to the gov. to show that the law would have been enacted without that factor. - Arlington Heights v. Metro. Housing Develop.court denied a rezoning request which would have allowed building of multi-family units fro low and moderate income tenants. The court refused to find an equal protection violation based on the lack of discriminatory intent. o Determining the intent of the official action can be difficult (outside of rare cases where racial discrimination is obvious on the face), and the court suggested that a fact intensive balancing test considering many factors including but not limited to: 1) the impact of the challenged decision (whether it disproportionately impacted one race); 2)the historical background of decisions under the official action, particularly if unequally applied in situations involving race; 3) the specific sequences of events leading up to the decision challenged in the case, including departures from normal procedures in making decisions and substantive departures, (i.e. the decision maker would have made a different choice had the applicant been white; race was the deciding factor); and 4) the legislative history where there are contemporary statements made by the governmental body who created the official action.

Affirmative Action Discuss the legitimacy of using race, ethnicity, or gender as a criteria for preferring one candidate over another. A diverse student body is a compelling state interest, but college and universities must consider each applicant individually. Race can only be a factor when defining admission criterion. I fit is the defining characteristic, violation of EP EP standardStrict scrutinymust be narrowly tailored to further a compelling gov. interest.

54

Regents of the University of California v. Bakkeheld that the admissions program violated the EP clause because it discriminated based on race. The program was designed to ensure the admission of a specified number of students from certain disadvantaged minorities. There was 2 different program, two admission committees. At least 16 spaces was to be filled by minority students, they could also compete for admission in the regular admission process. Court stated that it discriminated based on race, the use of a quota system violate EP. No compelling interest to justify classification. Grutter v. Bollingerthe court held that the use of race in admission decisions did not violate EP. Stating that using race to obtain a diverse student body is a compelling gov. interest. The law school was not aiming at obtaining a specific percentage of a particular ethnic group, which would have been unconstitutional racial balancing. Rather it was trying to capture a critical mass sufficient to generate the educational benefits that arise from a diverse student body. The interest was student body diversity "in the context of higher education," and was not focused on race alone but encompassed "all factors that may contribute to student body diversity. The entire gist of the analysis in Grutter was that the admissions program at issue there focused on each applicant as an individual, and not simply as a member of a particular racial group Gratz v. Bollingerseparate admission tracks are unconstitutional, quota systems are unconstitutional. The court struck down a undergraduate admission program. Students had to receive 100 pts to gain admission, student who qualified as underrepresented minorities received an additional 20 points. Parents involved in Community Schools v. Seattle--prohibited assigning students to public schools solely for the purpose of achieving racial integration and declined to recognize racial balancing as a compelling state interest.[1] In a 5-4 opinion delivered by Chief Justice John Roberts, five justices held that the School Boards did not present any "compelling state interest" that would justify the assignment of school seats on the basis of race. Associate Justice Anthony Kennedy filed a concurrence that presented a more narrow interpretation, stating that schools may use "race conscious" as a means to achieve diversity in schools but that the schools at issue in this case did not use a sufficiently narrow tailoring of their plans to sustain their goals.

Employment Affirmative Action Is the presumption of disadvantage based on race alone, and consequent allocation of favored treatment, a discriminatory practice that violates the equal protection principle embodied in the Due Process Clause of the Fifth Amendment? Yes. Overruling Metro Broadcasting (497 US 547), the Court held that all racial classifications, whether imposed by federal, state, or local authorities, must pass strict scrutiny review. In other words, they "must serve a compelling government interest, and must be narrowly tailored to further that interest." The Court added that compensation programs which are truly based on disadvantage, rather than race, would be evaluated under lower equal protection standards. However, since race is not a sufficient

55

condition for a presumption of disadvantage and the award of favored treatment, all race-based classifications must be judged under the strict scrutiny standard. Moreover, even proof of past injury does not in itself establish the suffering of present or future injury. The Court remanded for a determination of whether the Transportation Department's program satisfied strict scrutiny. - Adarand Const. v. Pena--Adarand, a contractor specializing in highway guardrail work, submitted the lowest bid as a subcontractor for part of a project funded by the United States Department of Transportation. Under the terms of the federal contract, the prime contractor would receive additional compensation if it hired small businesses controlled by "socially and economically disadvantaged individuals." [The clause declared that "the contractor shall presume that socially and economically disadvantaged individuals include Black Americans, Hispanic Americans, Native Americans, Asian Pacific Americans, and other minorities...." Federal law requires such a subcontracting clause in most federal agency contracts]. Another subcontractor, Gonzales Construction Company, was awarded the work. It was certified as a minority business; Adarand was not. The prime contractor would have accepted Adarand's bid had it not been for the additional payment for hiring Gonzales.

Equal Protection: Insular Minorities Illegitimate children, aliens, old age, mentally retarded. The law has not recognized the poor as a insular minority, the court has given heightened scrutiny on laws that deny fundamental rights based on wealth such as voting.

Alien Scrutiny classifications based on alienage varies with several factors. The court only scrutinize discrimination against resident aliens but not discrimination against illegal aliens. The state subjects discrimination against aliens by state or local government authorities to higher degree of scrutiny than it applies to discriminate against aliens by federal government. Even in the context of state and local governments, the limitations on application of strict scrutiny to classification based on alienage is the government function exception. - State gov.legal alienstrict scrutiny (unless government function exception) o State/local laws are subject to SS if based on alienage. i.e. welfare benefits, civil service jobs, etc. o The gov. function exception applies to laws that exclude aliens from positions immediately related the gov. function. Trigger rational level review - State gov.illegal alienrational level review - Federal gov.while the court applies SS to state provisions that discriminate against aliens, it has declined to extend it to the federal gov. because the power the federal gov. has over immigration and naturalization. (rational level review)

56

Gov. Function Exception: A classification based on Alienage must satisfy 2 prongs in order to fall within exception: 1. A classification that falls with the exception is valid if it bears a rational relationship to this legitimate gov. interest. - The court has upheld under the political function exception, laws that exclude aliens from becoming police, probation officers, as well as laws that denied aliens the chance to teach in public schools unless they declared their intent to become citizens. States (gov. function exception) Ambach v. NorwickP eligible for citizenship, but had refused to apply. Both had applied for certification as public school teachers in New York State. New York law prohibited the certification of non-citizen teachers who had not sought citizenship. Both applications were denied certification solely on that ground. Court held that statute did not violate EP because the state has a legitimate interest in futhering the educational goals and bear a rational relationship to the interest. It bars only aleins who have demonstrated an unwillingness to obtain citizenship. State (illegal aliens) Plyer-DoeTexas law denied free public education to undocumented children violated the equal protection clause. The EP clasue applied to these children. While undocumented aleins were not a suspect class, the law discriminated against innocent children. Although education is not a fundamental right, deprivation of education would place children at a lifelong disadvantage. The law was not rationally related to a legitimate public purpose. Rational Basis standard Federal Power (Aliens) Mathews v. Diazcongress can condition an aliens eligibility for participation in a federal medical insurance program in continuous residency requirement for 5 years and admission for permanent residency. The court held that it was permissible because fed. Gov. have the authority to regulate immigration and naturalization and it is reasonable for congress to make aliens eligibility depend on the character and duration of residency.

Illegitimate Children Intermediate Scrutinythe laws that treat illegitimate children different from legitimate children must be substantially relation to a important state interest. EP requires that a statute placing a burden on illegitimate children in the furtherance of proper state objections must be more carefully tuned to alternative considerations. - Trimble v. Gordon(a state cannot absolutely exclude illegitimate children from their fathers estate)--law allowed illegitimate children to inherit by intestate successions from their mothers, but not from their fathers. The legitimate the child, the law required the putative father to acknowledge the child and marry the mother. When the father died, he

57

was barred from inheritance because his father did not marry the mother although he established paternity. The statue was held that promoting legitimate family relationship was an important interest but a violation of EP because there was no relationship between the purpose and the statute. The court found an interest in the just and orderly disposition of property but the statute was not carefully tuned to alternative considerations. Lalli v. Lalli( a state can require that the paternity of the father be proved before the fathers death)the court upheld a statute that required an illegitimate child to obtain an order of affiliation declaring paternity in order to succeed by intestate succession. The order had to be issued in a proceeding commenced while the childs mother was pregnant or within two years of birth. The child could not produce an order, but presented other evidence that the decedent was his father. The court held that the statute was substantially related to the states interest in orderly distribution of the estate. It was distinguished because it did not require the parents to marry.

Sex (Gender) The court instituted a standard, dubbed "intermediate scrutiny", whereby the state must prove the existence of specific important governmental objectives, and the law must be substantially related to the achievement of those objectives. Important Government interest Th justify the categorization based on gender, the government may not rely on overbroad generalizations about male/female characterizations. - Frontiero v. Richardsonthe court invalidated a federal statute that allowed men in the service to claim their wives as dependant, but required service women in the service to make a showing that their husbands were dependent. The gender classification was based on generalizations about men and women. The court found the military's benefit policy unconstitutional, because there was no reason why military wives needed benefits any more than similarly situated military husbands. The Air Force argued that the policy was intended to save administrative costs by not forcing the military to determine that every wife was in fact a dependent. o This court employed strict scrutiny, that was not later adopted in Craig v. Boren - Craig v. Boran--was the first case in which a majority of the United States Supreme Court determined that statutory or administrative sex classifications had to be subjected to an intermediate standard of judicial review. Oklahoma passed a statute prohibiting the sale of "non-intoxicating" 3.2 % beer to males under the age of 21 but allowed females over the age of 18 to purchase it. The statute was challenged as Fourteenth Amendment Equal Protection violation. The Court held that the gender classifications made by the Oklahoma statute were unconstitutional because the statistics relied on by the state were

58

insufficient to show a substantial relationship between the statute and the benefits intended to stem from it. - Mississippi University for Women v. Hogan--The court held that the single-sex admissions policy of the Mississippi University for Women violated the Equal Protection Clause of the Fourteenth Amendment to the United States Constitution. The case failed intermediate scrutiny The court, while recognizing that the State has a significant interest in providing educational opportunities for all its citizens, found that the State failed to show that providing a unique educational opportunity for females but not for males had any substantial relationship to that interest. See United states v. virginia--is a case in which the Supreme Court of the United States struck down the Virginia Military Institute's long-standing male-only admission policy. - United States v. Virginiaa male only military school violated the EP clause because it excluded women. They attempted to establish a parallel school for women with substantially comparable benefits. The Supreme Court held that still, the exclusion of women from the mens academy violated EP because it deprived the women of the unique opportunities and its prestige. - laws that allows unwed mothers but not unwed fathers to stop adoption proceedings are invlaid Discrimination upheld - Military Draftthis case arose in the context of Congress authority over national defense system and military affair. In regards to women being drafted, congress determined that if the draft takes place, the need would be for troop combat and women are not eligible for combat. The restriction on women in the combat is a statutory restriction as a matter of public policy. Therefore, men and women are not similarly situated for the purpose of the draft. It did not violate Due Process. - A law granting automatic US citizenship to nonmarital children born abroad to American women, but requiring fathers of children born abroad to take specific steps to establish paternity-promotes important government interest of avoiding proof of paternity problems. De Jure/De Facto Discrimination "When a statute gender-neutral on its face is challenged on the ground that its effects upon women are disproportionably adverse, a twofold inquiry is thus appropriate. The first question is whether the statutory classification is indeed neutral in the sense that it is not gender-based. If the classification itself, covert or overt, is not based upon gender, the second question is whether the adverse effect reflects invidious gender-based discrimination. The challenger must show that the law reflects a purpose to discriminate on the basis of sex. - Personal Administrator of Massachusetts v. Feeney--upheld the constitutionality of a state law giving hiring preference to veterans over non-veterans.[1] The law was challenged as violating the Equal Protection Clause of the Fourteenth Amendment to the

59

United States Constitution by a woman, who argued that the law discriminated on the basis of sex because so few women held veteran status. The appellee... has simply failed to demonstrate that the law in any way reflects a purpose to discriminate on the basis of sex. Age Classifications based on age receive rational basis test. Age is not a suspect class. A has not experienced a history of purposefully unequal treatment. Additionally age is not a discrete and insular group in need of protection from the majority because everyone will be a member of that group if he/she lives a normal life expectancy. - the court upheld a law requiring mandatory retirement of all uniform state police officers a the age of 50. The rational level review standard was used. A legislative classification is subject to strict scrutiny only when the classification interfered with a fundamental right or disadvantaged a suspect class. The right to government employment was not fundamental and age was not a suspect class. Wealth (EP for the Poor) Rational Level review. Poor is not regarded as a discrete an insular minority. Laws that discriminate based on wealth are not entitled to the heighten scrutiny. However, regulations affecting the poor sometimes involve the fundamental strands of EP. Economical and Social Policy. - Dandridge v. Williamsthe court declined to extend strict scrutiny to a Maryland law that set a maximum amount of federal aid to families to AFDC. It commuted the standard of need for each eligible family based on the number of children. The regulation states that the family may not get more than $250 per month. But most of the larger families standards of need substantially exceeds the maximum granted by the state. Brought EP suit on groups that the limitation operates to discriminate against them merely because of the size of their family. The court held that this regulation deals with state regulation of social and economic field, not affecting any freedoms guaranteed by the Bill of Rights. No violation of EP merely because the classification made by the laws are imperfect. Public Education Wealth is not a suspect class and education is not a fundamental right - San Antonio Ind. Sch. Dist. V. Rodriguezthe court examined the method of funding schools in texas. The school system paid into the state fund based on property values. The funding in poorest and richest school districts differed hugely. Poor district received $356 per pupil, and richest received $549 per pupil. The funding district discriminated against poor persons whose income fall below identifiable level, those who are relatively poorer than others, and against those who lived in poor school districts regardless of income. The court held that a school-financing system based on local property taxes was not an

60

unconstitutional violation of the Fourteenth Amendment's equal protection clause. Education was neither explicitly or implicitly protected in the Constitution. He also found that Texas had not created a suspect class related to poverty. These two findings allowed the state to continue its school financing plan as long so it was rationally related to a legitimate state interest. Mentally Retarded Rational Level review. Mentally retarded is not a quasi-suspect class subject to intermediate scrutiny. - Cleburne Living Center, Inc. (CLC) submitted a permit application seeking approval to build a group home for the mentally retarded. The city of Cleburne, Texas refused to grant CLC a permit on the basis of a municipal zoning ordinance. CLC then sued the City of Cleburne on the theory that the denial of the permit violated the Equal Protection rights of CLC and their potential residents. Applying rational basis review the U.S. Supreme Court struck down the ordinance as applied to CLC. The Court declined to rule that the mentally retarded were a quasi-suspect or suspect class. The Court held that the denial of the permit was based on irrational prejudice against the mentally retarded and hence was invalid under the Equal Protection Clause of the Fourteenth Amendment. - The Court declined to grant the mentally retarded status as a suspect or quasi-suspect class because they are a "large and diversified group" amply protected by state and federal legislatures. Therefore, any legislation that distinguishes between the mentally retarded and others must be rationally related to a legitimate government interest in order to withstand equal protection review.

Fundamental Rights
Fundamental rights are protected under the Constitution. If they are denied to everyone, it is a substantive due process problem. If its denied to certain individuals, it is a EP problem. To be valid the action must be necessary and have a compelling interest. Access to the courts/Justice system In Griffin v. Illinois, the court held that indigent defendants be provided a trial transcript if necessary to for an appeal. The court extended Griffin in Douglas v. California holding that the 4th amendment required states to provide counsel to indigent defendants for their first appeal as a right. - Griffin v. Illinois--Black held that while the state court was not required by the federal constitution to provide appellate courts or a right to appellate review, because the state did grant appellate review at all stages of the proceedings, the Due Process and Equal Protection Clause protected the prisoners from invidious discriminations. The court held that destitute defendants must be afforded as adequate appellate review as defendants

61

who had money enough to buy the transcripts. It was held that the due process and equal protection clauses of the Fourteenth Amendment were violated by the state's denial of appellate review solely on account of a defendant's inability to pay for a transcript. California v. Douglas--Where the merits of the one and only appeal an indigent has as of right are decided without benefit of counsel in a state criminal case, there has been a discrimination between the rich and the poor which violates the Fourteenth Amendment. The Court held that a procedure like the one used by the state appellate court in which an indigent defendant was denied counsel on appeal unless he first made a preliminary showing of merit did not comport with fair procedure. In vacating, the judgment of the state appellate court, the Court stated that where the merits of the one and only appeal an indigent had as of right were decided without benefit of counsel, an unconstitutional line had been drawn between rich and poor.

Voting The court has given strict scrutiny to laws that makes it difficult to exercise voting rights or that dilutes the value of the vote. Voting is a fundamental right because it preserves other basis civil and political rights. It requires that each individual, not each group, have equal say in electing representatives. Restrictions on voting, other than the basis of age, redifency, citizenship, and invalid unless it passes strict scrutiny. Poll tax - Harper v. Virginia State Board of Electionsthe court invoked the EP to invalidate a law requiring voters to pay a $1.50 poll tax as a prerequisite for voting in state elections. Apply strict scrutiny, the court found that the poll tax did not further the states asserted interest in encouraging citizens to vote intelligently. Voter qualifications have no relation to wealth. Dilution of vote - Reynolds v. Sims--ruled that state legislature districts had to be roughly equal in population. challenged the apportionment of the Alabama Legislature. The Alabama Constitution provided that there be at least one representative per county and as many senatorial districts as there were senators. Ratio variances as great as 14 to 1 from one senatorial district to another existed in the Alabama Senate (i.e., the number of eligible voters voting for one senator was in one case 14 times the number of voters in another). o When gerrymandering has the purpose and effect of imposing burdens on a disfavored party, the 1st may supply better protection that EP. o When this occurs must redistrict by all deliberate speed. Must be done, at a minimum every 10 years Mathematical Precision - Karcher v. Daggett--Democrats in control of the New Jersey Legislature designed a plan for congressional redistricting in the state which the outgoing Democratic governor

62

signed into law. Even though the district populations differed by less than one percent from each other, they were clearly drawn to maximize Democratic power in the state. Even though the population differences in the districts were slight, the Court held that they were unconstitutional because they "were not the result of a good-faith effort to achieve population equality." Mahan v. Howell--In 1971, the Virginia legislature reapportioned itself. The plan for the House of Representatives provided for 100 representatives from 52 districts with each House member representing an average of 46,485 constituents(with a variance between largest and smallest being 16.4 percent, compared to the ideal 3.89 percent). Henry Howell challenged the plan as unconstitutional because its population deviations were too large to satisfy the principle of "one person, one vote." The Court found that the plan was constitutional under the Equal Protection Clause. The Virginia plan is not to be judged by the more stringent congressional standards in Section 2 of Article I. The Equal Protection Clause requires a state to make an "honest and good faith effort" to construct districts of as nearly equal population as practicable. Some deviations from the equal population principle are valid if based on legitimate considerations of a "rational state policy." The Virginia plan advanced the policy of reapportionment without sacrificing substantial equality.

Voting Based on Property Qualifications As long as the election in question is not one of special interest, any classification restricting the franchise on grounds other than residence, age, and citizenship cannot stand unless the state/district can demonstrate that the classification serves a compelling interest. - Hills v. StoneTexas has a dual election box. Those who rendered property taxes votes in one box. Those who had not rendered in another box. The bond will be deemed to have passed only if approved by a majority of the voters in the rendered box and majority between both boxes. The court held that it was unconstitutional because it created a classification based on rendering and disenfranchised those who did not render. No compelling interest. It was a general election. Not a special interest election. - Special interest elections can discriminatea water district was created for the purpose of disturbing agricultural water in which votes were allocated according to the access value for each voters land. Because the election had a special limited purpose the court held that the water district election was of sufficient special interest to a single group.

Access to the Ballot A state can have a statewide requirement of the number of signatures needed to be placed on a ballot, however, they cant create a classification. - Illinois State Bd. of Elections v. Socialist Workers PartyIllinois law required new political parties and independent candidates to obtain a different amount of signatures

63

than other political parties to be placed on the ballot. The court held that it was unconstitutional because the freedom to associate with a political party is a fundamental right, therefore restriction of access restricts the right of voters to choose a political candidate. The state must possess a compelling reason for classification. o Compelling purposeslimits ballot space to serious candidates, need to ensure elected person is the choice of the majority, need to show significant support Williams v. RhodesOhio made it virtually impossible for a new political party to be placed on the ballot. They required earlier deadlines to obtain signatures, they needed 5% more than dem. And rep. to be placed on the ballot. Court held it to be unconstitutional because the court had no compelling interest to justify the burdens placed on minor parties. Jenness v. FortsonGeorgia provided that a political candidate who received at least 20% of votes from the last election could have the candidates name placed on the ballot, but a non party candidate had to secure signatures of 5% of total number of electors eligible to vote in the last election to get name on ballot. The court held that the requirement was a reasonable way of serving the states interest in preventing ballot confusion.

Counting votes Bush v. Gorethe Supreme Court reversed the decision of the Fl. Sup. Ct. allowing several florida counties to recount ballots for the 2000 presidential election holding that the lack of uniform standards for determining voter intent violated the EP clause. - Article 2 of the Constitution affords a state legislature plenary power to select the manner of appointing electors. Once the state vests it power to select the president in its citizens, to be consistent with EP, a state cannot value one persons vote over another.

Right to Travel
There is a fundamental "right to travel" from state to state. Although the Constitution does not mention the right to travel, it is implied by the other rights given in the Constitution. (Although the right was recognized under the Equal Protection clause in this case, pre-Fourteenth Amendment, the right to travel was understood as protected by the Privileges and Immunities Clause (Article IV), as a privilege of citizenship, and therefore might have been applied to the states under the Privileges or Immunities Clause of Amendment XIV. Congress can not authorize a state to violate the 14th amendment. The right to travel have 3 components: 1. The right of a citizen to enter and leave one state 2. The right to be treated as a welcomed visitor and not an unwelcomed alien 3. And for those who elect to become citizens of another state to be treated like the other citizens

64

Waiting periods Where the state establishes a waiting period in order to receive benefits, it is likely invalid absent compelling interest. - Shapiro v. Thompsoncourt held that it was unconstitutional to limit welfare assistance to persons who lived in the state for a least a year. The stated reasoned, people who would be a lifetime burden on the state for public assistance deterred them from entering the state, conserve more resources for long term residents. The court held that the purpose imposed an unreasonable burden on the right to travel. The state failed the compelling interest test. - Saenz v. Roecourt struck down a California statute that denied new residents the same level welfare benefits as those who had been in California for more than 1 year. They would be limited to the amount of benefits they would receive in their home state for that year. Violation for EP and Privilege and Immunities clause Distinctions between old and new residents (doesnt pass rational basis test - Zobel v.WilliamsAlaska enacted a dividend program to distribute annual portion of funding to each state resident. The amount you receive was based on how long youve been a resident of the state. The challenge to the enactment was that it violated EP because they did not enjoy the same rights as long-term citizens. It violated EP because it created a permanent fixture based on length of residency and thus benefits were distributed unequally. The enactment could not pass the minimal level of scrutiny - Cannot require a one year residency requirement as a condition to receiving nonemergency hospitalization or Medicare at the countys expense. It is unconstitutional because it restricts interstate migration. Other restrictions - Tuition Paymenta state may charge lower tuition for instate residents. That out of state residents have to live in the state for a year to prior to getting in state tuition rates. This is allowed because once cannot equal the value of education with the necessities of life such as food, clothing, shelter. The durational requirements for attendance at a publically financed university does not involve such risk. - A state can restrict the right to vote to bona fide resident and can require a durational requirement. The interest penalizes two fundamental rights, the right to travel and the right to vote, so the lengthy durational requirement did noy serve any compelling state interest. 3o day restriction is reasonable. Foreign Travel The right to interstate travel is virtually unlimited, the right to international travel was a liberty interest protected by the DPC and could be regulated accordingly.

65

Haig v. Agee--involving Congressional delegation of authority over control of passports and the right to international travel. The Ex CIA agent had been traveling all over the world exposing CIA operatives. The United States Secretary of State at the request of the President revoked Agee's passport in 1979. Agee sued, alleging the secretary had no such authority, had denied him procedural due process rights, his substantive due process "liberty" right to travel under the Fifth Amendment, and had violated his First Amendment right to criticize government policies. The revocation of his passport did not impermissibly burdens his freedom to travel because the freedom to travel abroad with a "letter of introduction" in the form of a passport issued by the sovereign is subordinate to national security and foreign policy considerations; as such, it is subject to reasonable governmental regulation. The action was not intended to penalize his exercise of free speech and deter his criticism of Government policies and practices because assuming, arguendo, that First Amendment protections reach beyond our national boundaries, revocation of Agee's passport rested in part on the content of his speech. To the extent the revocation of his passport operates to inhibit Agee, "it is an inhibition of action," rather than of speech. And that failure to accord him a prerevocation hearing did not violate his Fifth Amendment right to procedural due process because when there is a substantial likelihood of "serious damage" to national security or foreign policy as a result of a passport holder's activities in foreign countries, the Government may take action to ensure that the holder may not exploit the sponsorship of his travels by the United States.

Privacy rights (Liberty Interest)


Privacy and Sexual Autonomy There is a 14th Amendment right of privacy through DP. Right to marriage, sexual relations, abortion, childrearing, are fundamental rights protected by the 14th amendment DP. In order to be upheld, the law has to be necessary to a compelling intersest. Sterilization - Skinner v. Oklahomathe court invalidated a statute that allowed the state to sterilize habitual criminal guilty of committing three felonies. It was invalidated based on EP grounds, holding that the law required unequal treatment of similarly situated criminals. Even though persons who were sterilized were not a protected class for EP purposes, it violated EP on grounds with the respect to a fundamental right. Marriage and procreation are fundamental rights. Compulsory sterilization laws are held to strict scrutiny. The law was struck down because there was no slighest basis that crime is genetic, failed strict scrutiny test. - Buck v. Bellcourt upheld statute instituting compulsory sterilization of the unfit, mentally retarded for the protection of the health and safety of the state. The states

66

rationale was that mental deficiency will/ can be inherited by offspring and the offspring may become wards of the state. Balanced the common interest of the individual and common good of the state. Contraception - Griswold v. Connecticut---administers of planned parenthood was charged with violating a statute that prohibited giving contraception to married persons. The court held that the statute infringed on the privacy of the marital relationship. The law prohibited the use of contraceptives rather than regulating its manufacture or sale. You could only use it to avoid STD but not to prevent pregnancy. The right to marital privacy falls within the penumbra of the bill of rights. The law failed SS. - Eisenstadt v. Bairdestablished rights of unmarried people to use contraceptives on the same basis as married couples. The law prohibited unmarried persons from using contraceptive to prevent pregnancy, but may not use to prevent STDs. However, married persons were allowed to use to prevent pregnancy. It stood as the proposition that if the right to privacy means anything, it is the right of the individual, married or single, to be free from unwarranted government intrusion. (9th amendment is a singal to look beyond the text of eh constitution. Marriage - Loving v. Virginialaw made it a crime for whites to marry nonwhites. The court held that antimiscengenation laws violated the EPC and Due process. Restricts the freedom to marry based on race, and right to marry is a fundamental right. Abortions (Fundamental right to privacy SS) The right to privacy includes the right of women to have an abortion. But because the state has an compelling interest in protecting the life and health of the woman an health of fetus, the court applies an unburden test. - Roe v. Wadethe Texas statute made it a crime to procure or attempt to procure an abortion. The court held the right to choose abortion was a constitutional right to privacy which derived from liberty in the DPC. The court rejects the argument that the fetus was a person for purposes of 4th amendment protection. The constitution protected those who were already born, avoiding the question of when life begins which is not a constitutional law question. Pre-Viability Rule Before viability, the state may adopt regulations protectiong the mothers health and life of the fetus only if the regulation does not place an undue burden or substantial obstacle to the womans right to have an abortion.

67

1. If an undue burden exist, the law is invalid if its purpose and effect is to place a substantial obstacle in the path of a woman seeking an abortion before the fetus attains viability. 2. A state can require abortions to be performed by licensed physicians and may take measures to ensure that the womans choice is informed 3. 24 hour waiting period between time of informed consent and procedure is not an undue burden. 4. A state requiring a minor to obtain parental consent is not an undue burden if there is a bypass procedure where minor may obtain the abortion with the consent of a judge. 5. Spousal notification or consent is an undue burden 6. Partial-Birth abortion planno undue burdenmust be allowed for the protection of both the mothers health and life. Post-viability abortions A state may prohibit abortions unless the womens life or health is threatened. Once the fetus is viable, the state has an interest in the life of the fetus and can override a womens right to choose abortion. However, if the womens health is at issue, the state cannot prevent the abortion. Cases - Doe v. Boltonupheld a provision allowing the physician to perform an abortion only if necessary based upon his clinical judgement. The court invalidated three procedural demands. 1. That the abortion be performed in a hospital accredited by JCAH 2. That the procedure be approved by the hospital committee staff 3. And physician determination be conferred by an independent examination of the patient by two other licensed physicians. - The EP clause do not require a state to pay the expense incident to a non-medical necessary abortion for indigent women simply because it had made the decision to pay the expenses incident to child birth. A state can make a value judgment favoring child birth over abortion implementing that judgment in favor of public funds. - The court upheld the Hyde amendment which permitted federal funding to reimburse the cost associated with child birth, but prohibited the use of federal funds to reimburse cost of abortions excepted under specific circumstances such as risk of danger to the mother, rape, or incest. - Court upheld regulation that limited the ability of recipients of federal/state funds to engage in abortion related activity. - PP v. Casey--The plurality asserted that the right to abortion is grounded in the Due Process Clause of the Fourteenth Amendment. The plurality then overturned the strict trimester formula used in Roe to weigh the woman's interest in obtaining an abortion against the State's interest in the life of the fetus. The plurality recognized viability as the point at which the state interest in the life of the fetus outweighs the rights of the woman

68

and abortion may be banned entirely "except where it is necessary, in appropriate medical judgment, for the preservation of the life or health of the mother".The plurality also replaced the heightened scrutiny of abortion regulations under Roe with a lesser "undue burden" standard . A legal restriction posing an undue burden was defined as one having "the purpose or effect of placing a substantial obstacle in the path of a woman seeking an abortion of a nonviable fetus." Partial Birth Abortions - The court struck down a statute that prohibited any partial birth abortions unless that procedure was necessary to save that persons life because the legislation lacked the requisite exception for the life and the health of the mother. It completely disregarded the heath exception. It placed an undue burden on a woman seeking a pre-viability abortion. The state can regulate abortions post viability except where it is necessary for preservation for the life and heath of the mother. - The court upheld Congress's ban and held that it did not impose an undue burden on the due process right of women to obtain an abortion. majority opinion held that "ethical and moral concerns", including an interest in fetal life, represented "substantial" state interests which (assuming they do not impose an "undue" burden) could be a basis for legislation at all times during pregnancy, not just after viability.

Homosexuality - Bowers v. Hardwickupheld, in a 5-4 ruling, the constitutionality of a Georgia sodomy law criminalizing oral and anal sex in private between consenting adults when applied to homosexuals. Argued that the Constitution did not confer "a fundamental right to engage in homosexual sodomy. Police arrested a man in his bedroom for committing an act that violated the Georgia sodomy laws with another adult male. Although the statute extended to heterosexual activity and to married individuals, the court limited its decision to homosexual activity. - Romer v. Evans--Court had ruled that a law criminalizing homosexual sex was constitutional. An amendment to the Colorado state constitution ("Amendment 2") that would have prevented any city, town or county in the state from taking any legislative, executive, or judicial action to recognize gay and lesbian citizens as a protected class was passed by Colorado voters in a referendum. A state trial court issued a permanent injunction against the amendment, and upon appeal, the Colorado Supreme Court ruled that the amendment was subject to "strict scrutiny" under the Equal Protection Clause. The state trial court, upon remand, concluded that the amendment could not pass strict scrutiny, which the Colorado Supreme Court agreed with upon review. Upon appeal to the United States Supreme Court, the Court ruled in a 6-3 decision that the amendment did not even pass the rational basis test, let alone strict scrutiny.

69

Lawrence v. Texasthe court invalidated a Texas law that made it a crime for two persons of the same sex to engage in intimate sexual conduct. The majority held that intimate consensual sexual conduct was part of the liberty protected by substantive due process under the Fourteenth Amendment. Lawrence invalidated similar laws throughout the United States that criminalized sodomy between consenting adults acting in private, whatever the sex of the participants.

Right to Die
The right to refuse medical treatment is a part of an individuals liberty interest protected by the 5th and 14th amendment DPC. Cruzan v. Missouri Dept. of HealthCruzan sustained serious injuries in an automobile accident that left her in a permanent veg. state. Her parents sought a court order allowing them to remove the artificial feeding tube in her stomach that would cause her death. The court held that there was a constitutional right for the patient herself to refused treatment. The court stated that there must be clear and convincing evidence that the patient, herself would choose to refuse treatment. Washington v. Glucksberg-- held that a right to assistance in committing suicide was not protected by the Due Process Clause. The Court asserted that because assisted-suicide is not a fundamental liberty interest, it was therefore not protected under the 14th Amendment. Assisted suicide is not an interests "deeply rooted in the nation's history" do not qualify as being a protected liberty interest. Vacco v. Quill--It ruled that a New York ban on physician-assisted suicide was constitutional, and preventing doctors from assisting their patients, even those terminally ill and/or in great pain, was a legitimate state interest that was well within the authority of the state to regulate. In brief, this decision established that, as a matter of law, there was no constitutional guarantee of a "right to die." that the judiciary must look to the Constitution, rather than to the stated "importance" of a right, when determining whether that right was, indeed, fundamental. Because New York's ban did not infringe upon a fundamental right, and because respondents were not claiming that the "right to die" was fundamental. the Court noted that the law plainly recognized the difference between "killing" and "letting die." It also recognized that the State of New York had, as a matter of policy, a compelling interest in forbidding assisted suicide, while allowing a patient to refuse life support was simply an act of protecting a common-law right.

You might also like